Sie sind auf Seite 1von 56

Author: Remi Malahieude

Based on: Release FI-A2-engb 2/2008 (1003) – Steve Elliot's Finance Notes from Oct. 30-2006 Date: January 23, 2010

Finance
Be aware that beyond the Past Papers, the Faculty Board will post some extremely valuable Revision Exercises
in the Faculty Download Area. Those files are not there from the start of term. The Professor will post them right
before the minor diet. I highly recommend doing all those prior to any online Case study or Past Paper.
Good luck in your finance journey.

Module 1
Finance is the economics of allocating resources across time.

Financial Markets
- Participation is financial markets is driven in part by the desire to shift future resources to
the present so as to increase personal consumption, and thus satisfaction.
- Or one may shift resources to the future by lending them, buying common stock, etc. In
exchange, they get an expectation of increased future resources, in the form of interest,
dividends, and/or capital gains.
- Where financial investments serve the purpose of reallocating the same resources across
time, real asset investment can actually create new future resources.
- The provision of funds for real asset investment is important, as is the
allocative information that financial markets provide to those interested in making real
asset investment.
- Financial markets can help tell the investor whether a proposed investment is worthwhile by
comparing the returns from the investment with those available on competing uses.
- Financial market participants are risk-averse, they would choose the less risky of two
otherwise identical investments.

Market Interest Rate & Prices


- The market interest rate is the rate of exchange between present and future resources.
Always positive because lenders keep their money if they are not offered a positive return (MCQ1B1Q1)
- Determined by the supply and demand of resources to be borrowed and lent.
- At any given time there are numerous market interest rates covering different lengths of
time and investment riskiness.
Market IR is but one of many interest rates (MCQ1B1Q1)

A Simple Financial Market

Shifting Resources in Time


- A financial exchange line is comprised of any transaction that a participant with an initial
amount of money may take by borrowing or lending at the market rate of interest.

- 1/56 -
- The line appears on a graph with CF1, the cash flow later on the vertical axis, and CF0, the
now cash flow on the horizontal.

- 2/56 -
- If a participant had $1000 at t0 and wished to borrow whatever he could with a promise to
repay $1540 at t1, how much could he borrow? Assume a 10% interest rate.
CF1 = CF0 (1+ i)
CF1 $1540
CF0 = --------- or ---------- = $1400
(1+ i) 1.10

The participant could borrow $1400 with a promise to repay $1540 at t1. The maximum
amount the participant could consume at t0 is thus $2400 (present wealth). $1400 is the
present value of $1540.
- Present value is defined as the amount of money you must invest or lend at the present
time so as to end up with a particular amount of money in the future.
- Finding the present value of a future cash flow is often called discounting the cash flow.
- Present value is also an accurate representation of what the financial market does when it
sets a price on a financial asset.
- Present value is the market value of a security when market interest rates or opportunity
rates of return are used as discount rates.
- Present wealth: total value of a participant’s entire time-specified resources.

Investing
- Investing in real assets allows for an increase in wealth because it does not require finding
someone to decrease their own.
- For wealth to increase the present value of the amount given up for real asset investment
must be less than the present value of what is gained from the investment.

Net Present Value


- The present value of the difference between an investments cash inflows and outflows
discounted at the opportunity costs (i) of those cash flows.
CF1
NPV = -------- – CF0
(1+ i)
- It is generally true that NPV = Change in present wealth.
- It is also the present value of the future amount by which the returns from the investment
exceed the opportunity costs of the investor.

Internal Rate of Return


- Calculates the average per period rate of return on the money invested.
- Once calculated, it is compared to the rate of return that could be earned on a comparable
financial market opportunity of equal timing and risk.
- IRR is the discount rate that equates the present value of an investment’s cash inflows and
outflows. This implies that it is the discount rate that causes an investment’s NPV to be
equal to zero. (MCQ1B2)
CF1
NPV = 0 = – CF0 + -----------
(1+ IRR)

CF1
(1+IRR) = --------
CF0
- An IRR greater than the financial market rate implies an acceptable investment (and a + NPV), an
IRR lower than it does not (and a –NPV).
- IRR and NPV usually give the same answer as to whether an investment is acceptable, but
often give different answers as to which of two investments is better.

Corporate Example
- The sole task of a company is to maximise the present wealth of its shareholders.
- A company would accept investments up to the point where the next investment would
have a –NPV or an IRR less than its opportunity cost.
- 3/56 -
- Although shareholders have different consumption preference across time, companies
should
1. Ignore the shareholders preferences
2. Allow the market to reallocate resources across time

1. because the financial markets allow shareholders to reallocate resources by borrowing


and lending as they see fit. This way allows the company to concentrate on maximising
shareholder wealth. (MCQ1B3)

More Realistic Financial Markets

Multiple Period Finance


- Multiple period exchange rates (interest) are written as (1 + in)n, where n is the number of
periods.

Compound Interest
- Compounding means that the exchange rate between two time points is such that interest
is earned not only on the initial investment, but also on previously earned interest. The
amount of money you end up with by investing CF0 at compounding interest is written
CF0 [1 + (i/m)]m t
where m is the number of times per period that compounding takes place, and t is the
number of periods the investment covers.
- The most frequent type of compounding is called continuous. Interest is calculated and
added to begin calculating interest on itself without any passage of time between
compoundings. This reduces the above formula to CF0(eit), where e is = 2.718…., the base
of a natural logarithm.
CF0 (eit) = Continuous compounding

Multiple Period Cash Flows


- To find the present value of a cash flow occurring at any one future time point the following
formula is used:
CFt
PV = ---------
(1+ it)t
- The present value of a set (stream) of cash flows is the sum of the present values of each
of the future cash flows associated with the asset, calculated:

CF1 CF2 CF3


PV = --------- + --------- + --------
(1+ i1)1 (1+ i2)2 (1+ i3)3

Multiple Period Investment Decisions


- Calculating NPV when the investment decision will affect several future cash flows must
include all present and future cash flows associated with the investment.
CF1 CF2 CF3
NPV = CF0 + --------- + --------- + --------
(1+ i1)1 (1+ i2)2 (1+ i3)3
- Calculating the IRR of a set of cash flows involves finding the discount rate that causes
NPV to equal zero;
CF1 CF2 CF3
NPV = 0 = CF0 + ------------ + ------------ + -----------
(1+ IRR)1 (1+ IRR)2 (1+ IRR)3
- The only way to solve for the IRR of a multiple period cash flow stream is with the trial and
error technique.

- 4/56 -
Calculating Techniques and Short Cuts in Multiple Period Analysis
- The instruction to calculate the PV of a stream of future cash flows is;
T CFt
PV = Σ --------- (1.2)
t=1 (1+ it)t
- When discount rates are consistent across the future this changes to;
T CFt
PV = Σ -------- (1.3)
t=1 (1+ i)t

Calculation Methods
- Start with the CF furthest from the present, discount it one period closer and add the CF
from the closer time point, discount that sum one period nearer, etc. Continue process until
all cash flows are included and discounted back to t0. PREFERRED METHOD
- Use present value tables. Adding up the cash flows after discounting each one for its
respective time period.
- Tables are valuable when finding the present value of annuities. A constant annuity is a set
of cash flows that are the same amount across future time points.
- A perpetuity is a cash flow stream assumed to continue forever. Formula is simply a
division of the constant per-period CF by the constant per-period discount rate:
CF
PV = -------- (1.4)
i
- A slight modification of the above allows for the assumption that the cash flows will
continue forever, but will grow or decline at a constant percentage rate during each period
(AKA growth perpetuity),
CF
PV = -------- (1.5)
(i – g)
where g is the constant per-period growth rate of the cash flow.

- This equation will not work when i<g.

Interest Rates, Interest Rate Futures and Yields


- Interest rates that begin at the present and run to some future time point are called
spot interest rates.
- The set of spot rates in a financial market is called the term structure of interest rates.
- A coupon bond has a face value that is used, along with its coupon rate, to figure a pattern
of cash flows promised by the bond.
- These cash flows comprise interest payments each period which continue until the maturity
period when the face value itself, as a principal payment plus the final interest payment, is
promised.

The Yield to Maturity (YTM)


- The YTM is the IRR of the bonds promised cash flows, or the average per period interest
rate on the money invested in the bond.
The set of YTMs is known as the yield curve (MCQ1B9)
- Bonds can have the same spot rates, cash-flow risk, and number of interest payments, yet
have different YTMs. This is caused by a difference in the cash-flow patterns of each bond.
- This is dubbed the coupon effect on the YTM. Named as such because the size of the
coupon of a bond determines the pattern of its cash flows, and thus how its YTM will reflect
the set of spot rates that exist in the market.
- It is unwise to make comparisons among securities on the basis of their YTMs unless their
patterns of cash flows (or coupons, for bonds) are identical.
- YTMs express both earning rates and the amounts invested across time. (MCQ1B9)
- Issue price = par value = face value

- 5/56 -
Forward Interest Rates
- Interest rates that begin at some time point other than the present (to).
- The amount invested in an asset across time can be found by accruing past invested
amounts outward at the same rates that were used to discount cash flows back in time.
- This amount allows for a calculation of a forward interest rate.
- Forward interest rates are usually indicated as ƒ with a left subscript indicating the rates
start time point and a right one indicating the ending time point. Therefore:
CF1 ( 1 + 1ƒ2 ) = CF2

( 1 + i2 )2 = ( 1 + i1 ) ( 1 + 1ƒ2 ) = ( 1 + 0ƒ1 ) ( 1 + 1ƒ2 )


( 1 + i3 )3 = ( 1 + i1 ) ( 1 + 1ƒ2 ) ( 1 + 2ƒ3 ) = ( 1 + 0ƒ1 ) ( 1 + 1ƒ2 ) ( 1 + 2ƒ3 )
( 1 + i4 )4 = ( 1 + i1 ) ( 1 + 1ƒ4 )3 = ( 1 + i3 )3 ( 1 + 3ƒ4 ) = ( 1 + 0ƒ3 )3 ( 1 + 3ƒ4 )

- This formula calculates the implied forward rate for a bond.


- If forward rates are known, the spot rate of interest can be found by multiplying together 1
plus each of the intervening forward rates, taking the n th root of that product (where n =
number of periods covered), and subtracting 1. (page 1/35)

Interest Rate Futures


- The futures market in interest rates allow you to avoid (or hedge) the risk that interest rates
might change unexpectedly (therefore, potentially, reducing an NPV to a point where an
investment is no longer worthwhile).
- Expectations around future interest rates and future values are almost never completely
accurate because, between the time the expectation is formed (t0) and the realisation
occurs (t1), additional information will have appeared that causes the market to revise its
cash flow expectations, its opportunity costs, or both.
- Financial futures markets allow participants to guard against this kind of risk by buying and
selling commitments to transact in financial securities at future time points – at prices fixed
as the present.
- Allows a guaranteed set of discount rates for an asset NPV by agreeing to sell securities at
set prices across an assets life.
- Hedging takes away both good and bad surprises.

Interest Rate Risk & Duration


- The variability of values due to changes in interest rates is the effect of interest rate risk.
- Duration is a kind of index that tells us how much a particular bond value will go up and
down as interest rates change.
- It is the number of periods into the future where a bonds average value is generated. The
greater the duration of a bond, the farther into the future its average value is generated,
and the more its value will react to changes in interest rates.
- Duration can be calculated by weighting the time points from which cash flows are
generated, by the proportion of total value generated at each time:

t x CFt t x CFt
t t
Duration = Σ (1+it)t = Σ (1+it)t
t=1 Bond value t=1 Bond NPV

- Duration is the starting point for an important aspect of professional bond investing called
immunisation.

- 6/56 -
Annualized growth rate calculation

Present Value PV = CF x PVIFi,n


PVIFi,n= PV Interest Factor [Table A2.1]

Present Value of Annuity PVA = CF x PVAIFi,n


PVAIFi,n = PV of Annuity Interest Factor [Table A2.2]

Time out: for e.g. Year 5-17: 17 – 5 = 13 years <- (5-1) = 4


PVA = CF (PVAIFi,17 – PVAIFi,4)

If annuity starts today PVA = CF x PVAIFi,n x (1+i)

Future Value FV = PV x FVIFi,n


= CF x FVIFi,n
= CF0 [1 + (i/m)]m t
= PV [1 + (i/m)]m t
FVIF = Future Value Interest Factor [Table A2.3]
FVIF = FV / PV

Future Value of Annuity FVA = CF x FVAIFi,n


FVAIFi,n = Future Value Annuity Interest Factor [Table A2.4]

YTM Calculation/ Interpolation: FAQ mini case 1.8 and 1.9:

r1 = fist YTM rate guess


N1 r2 = second guess
YTM = r1 + [ ---------- x (r2 – r1) ] N1 = NPV fist guess = I Bond value – First guess I
N1+ N2 N2 = NPV second guess = I Bond value – First guess I

Ignore negative sign for NPV, take absolute values

 RevEx1Q6: What us the return offered if invest 1200 now for 6 years and obtain 1900 at the end of that time?
FVIFr,n = FV / PV = 1900 / 1200 = 1.5833 (1+i)6 = 1.5833 => i = (1.5833)1/6 = 7.96%

 RevEx1Q27: What should you pay for an asset that produces a level of cash stream of $50,000 every 6
months for the next 10 years, if your opportunity cost of capital is 8% (annual rate), compounded per
period?
PVA = CF x PVAIFi,n [Table A2.2] = 50,000 x PVAIF4,20 = 50,000 x 13.59 = 679,500

 A cny offer to pay you 200,000 today, it will pay you 30,000 pa at the end of Y5 and then a further 12years.
If your cost of capital is 10%, would you accept?
 This is a 13yr annuity at 10% = 7.1034 [Table A2.2], PVA = CF x PVAIFi,n = 30,000 x 7.1034 = 213,100
CF at the end of Y5, the annuity bring the value back to the start of Y5, so discount rate end of Y4= 0.683
[Table A2.1] = 1/ (1.1)4 => PV = CF x PVIFi,n = 231,100 x 0.683 = 145,550 => don’t accept

or

 (17 year annuity – 4 year annuity) x 30,000 = 145,550 PVA = CF (PVAIFi,17 – PVAIFi,4)

- 7/56 -
 RevEx1Q30: 1m paid 100,000 pa starting today vs. a 700,000 one time payment today, which one do you
take? Opportunity cost is 8%
PVA = CF x PVAIF8%,10 periods [Table A2.2] x (1+r) [as the annuity starts today] = 100,000 x 6.7101 x 1.08 = 724,690.80

 MCQ1B5: A bond trade at $110.5 in the market, its coupon is 9%, which has just been paid and next
payment is exactly 1 year. T/he bond has 5 years until maturity (at $100). YTM?
9 9 9 9 109
110.5 = -------- + -------- + -------- + -------- + --------
(1+ r)1 (1+ r)2 (1+ r)3 (1+ r)4 (1+ r)5

 MCQ1B10: 7% coupon bond paying i semi-annually trades at $98.12 just after the interest payment. Bond
has 3 years maturity. YTM?
3.5 3.5 3.5 3.5 3.5 103.5
98.12 = -------- + -------- + -------- + -------- + -------- + --------
(1+ r)1 (1+ r)2 (1+ r)3 (1+ r)4 (1+ r)5 (1+ r)6

- 8/56 -
Module 2 – Fundamentals of Company Investment Decisions
The corporation (plc) is an organization that raises money from capital suppliers by issuing contracts
(securities), invests that money in productive assets, operates those assets (perhaps hiring other resources
such as management and labour), and distributes the money proceeds from operating those assets to all that
have claims on the proceeds.

Capital suppliers are those who own equity and debt claims on a company.

Investment Decisions & Shareholder Wealth


- When a company issues shares to raise money from the capital market it creates a security
known as one of the following: ordinary shares, common stock, equity, common shares,
ownership capital, etc.

Important things to remember about this type of capital claim:


1. It is a residual claim
- Equity has no specific contract with the company that requires any particular
amounts of money to be paid to shareholders at any particular time.
- Shareholders have only the entitlement to vote for the directors of the
company.
- Directors and management are agents of the shareholders maximising the
wealth of its current shareholders.
2. Equity has limited liability
- The possible losses that a shareholder can incur are limited to the value of the
shares that the shareholder owns.
3. Until all other contracts that the company has entered into have been filled, the
shareholders are entitled to nothing. Once met however, the shareholders have an
ownership claim on all of the remaining corporate resources.

- Shareholder wealth is the market value of the common shares or equity that the
shareholders own.
- If a company wishes to maximise its shareholders wealth, it should seek to maximise the
market value of the shareholders ordinary shares.

The Market Value of Common Shares


- is the discounted value of all future dividends that the current shareholders are expected to
receive.
Dividend1 + Value1 Div1 + E1 Div1 Div2 + E2
Value0 = ---------------------------------  E0 = ------------- , and E0 = ---------- + -------------
(1 + Equity discount rate) (1 + re) (1 + re) (1 + re)2
So true value:
Div CF
E0 = -------  PV = ------ (Perpetuity)
re i
Investment and Shareholder Wealth
- The use of a properly calculated investment NPV does in fact result in an increase in the
present wealth of shareholders.

Investment Decisions in All-Equity Corporations


When making an investment;
1. The total value of a company increases by the NPV of the investment plus what it cost to
undertake it.
2. The shareholders experience a wealth increase equal to the investment’s NPV.
3. Existing shareholders of the company get a wealth increase equal to the investment’s NPV
regardless of who contributes the money necessary to undertake the investment (i.e.
forgone dividend, new equity holders, or even the original holders).

Investment Decisions in Borrowing Corporations

- 9/56 -
- Corporate NPV is equal to the change in wealth of existing shareholders even if some of
the money for an investment comes from creditors/ bondholders instead of equity holders.

Share Values and Price/Earnings Ratios


- P/E ratios are offered as signals that the market is providing as to the company’s future
earnings prospects, growth rate, riskiness, etc.
- The P/E ratio is nothing more or less than the ratio between the present value of all the
company’s future dividends (its market price) and its expected earnings during the first
period.
- For certain companies with very simple cash-flow patterns (i.e. constant, or constant
growth perpetuities) we can derive a specific relationship between P/E ratios and equity
discount rates:
Dividend per share Equity per share
Price per share = ------------------------- = ----------------------
re re
So
Price per share 1
P/E = --------------------------- = -----
Earnings per share re

- For companies who are not paying out all earnings as dividends:

Dividend per share Earnings per share x payout ratio*


Price per share = ------------------------- = -----------------------------------------------
( re – g ) ( re – g )
So
Price per share 1
P/E = --------------------------- = ----------- x payout ratio*
Earnings per share ( re – g )

* where payout ratio is the percentage of earnings paid as dividend.


= dividend payout ratio

Dividend
Payout Ratio = ---------------
Earnings

- Caution must be exercised when comparing the P/E rations of different companies to
ensure that all but one of the factors influencing market prices are reasonably similar.

When a dividend is paid, the share priced falls by that amount e.g. share price = 100p, cny pays 10p dividend,
the share price falls to 90p. That dividend is pay out of the share price, it is history (FB/FAQ)

2003 2004 2005 2006 2007 2008 2009


RE2Q6: Dividend 5.8p 6 6.3 6.7 7.1 7.7 8.5 re=14%
What is the share price if you base the growth of dividends on (i) the entire period (ii) the last 2 years?
(i) g = (8.5/5.8)1/6 = 6.577% P0= (8.5 x 1.065777) / (0.14 – 0.06577) = 122p
(ii) g = (8.5/7.1)1/2 = P0= 203p

The DGM can be manipulated to give different valuations if you use different spreads of years to calculate the
growth rate of dividends. if g>r => multi-stage valuation model (RE2iQ8)

The share price is the discounted value of all future Dividends (FAQ/ FB )
- 10/56 -
(ie. the PV of all future dividends = value of the cny) MCQ2B1

A high P/E ratio usually signifies a cny that is growing fast. Or it might be a cny where profits have almost
disappeared. Or cny takeover target and shares have been bid up. IF so, they will usually keep most of their
earnings for reinvestment to continue that growth. More value slow growing cnies will pay larger dividends.

The payout ratio indicates how much a cny’s earnings are paid out in the form of dividends (the balance being
available for reinvestment). FB/FAQ

P/E ratio is of limited use. Compare similar cnies (same sector/business). Cnies should have same accounting
conventions => earnings based on the same calculations. Adjustments required/needed to deliver measure of
earnings close to cash earnings rather than accounting earnings.
 not across sectors -> meaningless. Accounting figures still subject to manipulation
(RE2iQ7)

You can’t have a high dividend payout ratio and a high growth rate (g). [FAQ]

- 11/56 -
Module 3 – Earnings, Profit, and Cash Flow
Total Corporate Value Change
- The increase or decrease in the market value of all of a corporation’s capital claims that
would take place if the investment were accepted.

Corporate Cash Flows – activity across time

- Financial cash flows are the cash amounts that are expected to occur at the times for which
the expectations are recorded.

Typical Cash Flows (Company with zero debt, 100% equity financed) (‘000s)
Now Yr 1 Yr 2 Yr 3
Customers 0 +17 500 +23 500 +4 000
Operations 0 -7 000 -3 830 -5 200
Assets -10 000 -4 000 -2 000 0
Government 0 -4 000 -8 085 +5 600
Capital* (FCF) -10 000 +2 500 +9 585 +4 400

2 500 000 9 585 000 4 400 000


NPV = -10 000 000 + -------------- + ------------- + ------------- = +3 500 000
(1.10) (1.10)2 (1.10)3

1. Customers – The amounts of cash expected to take in from sales and/or selling used assets.
2. Operations – Cash flows that are paid in cash that year, be deductible for taxes that year,
and not be a payment to a capital supplier.
3. Assets – While the cash flow is made at time listed, it is not deductible at that point and
must therefore be capitalised and depreciated across time.
4. Government – Taxes paid due to the investment.
5. *Capital – The amounts of cash that could be taken out of the corporation by it’s capital
suppliers and still have the investment run as planned (AKA Free Cash Flow). The value of
all future increases/decreases in dividends expected due to the project (because company
is equity financed).

Cash Flows and Profits


- Cash flows are not the same as he numbers that appear in financial statements of
corporations.
- One of the biggest differentiation is that accounting figures often report cash flows for time
periods other that that which the flow occurs.

- 12/56 -
- Stakeholders = Gouvernment, Employers, Lenders different than shareholders!!!!

- Land = Opportunity cost (a cash outflow), in a CF analysis, initial value of land is an


outflow, final value of land at the end of the project is an inflow.

- Salvage value is part of the depreciation calculation. An Asset is usually fully depreciated,
adjusting for any expected salvage value at the end of its useful life. [MCQ3B7]

- Debtors = Account receivables


The accounting term debtors will include customers who will not pay until the next period, sot it is wrong to
include it in a CF. [MCQ3A6]

- When constructing project CF figures, Interest payments are excluded because they are
not part of the investment decision (FCF*= FCF-ITS) [MCQ3A7]

- Depreciation is an expense for tax purpose. [MCQ3A10]

- Capitalisation allocates the cost of the asset over a number of time periods. [MCQ3B5]

- FCFs and Profits after tax of a project will add up to the same amount. [MCQ3B10]

- Profit = Income = Earnings

- Your have an accountant’s income statement for 2 projest, one purely equity financed, one
partly financed with debt, but identical in all other respects.
- The all equity finance project will report a higher income
Because in the accounting figures, interest is regarded as an expense, so the debt finance cny will have
higher expenses and so lower profit as a result (MCQ4A2)
- The debt financed cny will have a greater CF than the equity financed cny (MCQ4B2)

- The cash that debt suppliers expect to get in the future = in PV terms to the amount that is
raised from them at present, so debt’s wealth is unaffected by the investment. (MCQ4A3)

- The figures to use in the WACC calculation are market values of debt and equity.
(MCQ4A4)

- All equity CFs are the CFs that exclude the ITS. They are the CFs that would exist if the
project was only funded by equity. (MCQ4B3)

- The appropriate rate to discount all equity CFs in a WACC calculation is the tax shield
adjusted discount rate. (MCQ4B4)

- What do FCFs of an investment project which are appropriate to be discounted with the
project’s WACC include?
All changes in the payments between the firm and its capital suppliers induced by the project, were the
project to be financed only with equity.

- 13/56 -
Module 4 – Company Investment Decisions using the WACC
- The weighed average cost of capital (WACC) is a discount rate that combines the capital
costs of all the various types of capital claims that a company issues.

Free Cash Flow and Profits for Borrowing Corporations


- Financial markets place values on corporate debt claims by discounting with risk-adjusted
rates the amounts of cash that the company is expected to pay to those claims.
- Debt has a higher priority claim to cash than does equity.
- A partially debt financed investment, ceteris paribus, will have higher free cash flows than
one which is 100% equity financed.
- This is due to tax laws that allow interest on loans to be written-off.
- This is called a corporation’s income tax shield ITS.
- Income tax shields can be calculated by taking the expected interest payment on each
period and multiplying it by the corporate income tax rate.

Investment Value for Borrowing Corporations


- Debt suppliers wealth should be unaffected by the investment. Figured by taking the
interest + principal for each period and dividing by the debt interest rate.
- Equity wealth change is figured out by comparing the investment-induced change in equity
value to any foregone dividend. Value change is calculated by subtracting principal and
interest from the FCF for each period and dividing that by an appropriate discount rate.

Overall Corporate Cash Flows and Investment Value


- To figure out NPV as a whole, the FCFs must be discounted with a rate commensurate with
their risk.
- To find this discount rate the debt and equity rates must be combined in proportion to their
claims on the corporate cash flow. This value is given by the market value of the two
claims:

Debt market value Equity market value


Overall rate = ------------------------ x Debt required rate + -------------------------- x Equity required rate
Total market value Total market value

- Valuing the security is now easy:


n FCF
Value of the project = Σ ----------------------------------
t=0 (1 + overall required rate)t
Investment NPV and the WACC
- The above technique is the overall NPV method.
- Common practice in financial analysis of corporate investment is that when estimating the
cash flows of a project, its interest tax shields are not included in the cash flows.
- In order to calculate an accurate NPV using cash flows that exclude interest tax shields, the
effect must also be included in the discount rate.
- The WACC is the discount rate that:
a. Reflects the operating risks of the projects.
b. Reflects the projects proportional debt and equity financing with attendant financial risks.
c. Reflects the effect of interest deductibility for the debt-financed portion of the project.
- In order to reflect interest deductibility in the discount rate (WACC), the weighted average
must use the company’s after-tax cost of debt rather than the debt suppliers required rate.
- Cost of debt in a company with deductible interest is simply debts required return multiplied
by the complement of the corporate income tax rate:

Debt cost = Debt required return x (1 - Corporate income tax rate)

- WACC is therefore calculated as follows;


Debt market value Equity market value
WACC = ------------------------ x Debt cost rate + -------------------------- x Equity required rate
Total market value Total market value

- 14/56 -
- The WACC-NPV analysis of an investment project is performed by discounting the project’s
FCF, not including the interest tax shields that the projects financing will generate.
- This adjusts for the deductibility of corporate interest in the discount rate as opposed to the
CF of the project.
- Companies using the WACC-NPV are willing to specify the expected proportions of debt
and equity in terms of their market values, but they do not know exactly what the claims will
be worth until after the analysis is complete.

The Adjusted Present Value Technique


- APV does not require knowledge of debt/equity proportions, but does require that the
interest tax shields of the project be estimated.
- APV is therefore preferred by corporations who are comfortable in estimating the amounts
of debt the projects will use.
- If performed correctly both APV and WACC-NPV will give the same answers.
- APV finds the NPV by first finding the value of an investment as if it were financed only by
equity, and then adds the PV of the projects interest tax shields.
- All-equity value is calculated by adding all the CFs discounted by the (an) all equity
discount rate.
- Interest tax shields value is calculated by discounting the shield CFs by the risk-adjusted
rate for debt cash flows:
APV = (All-equity value) + (Interest tax shield value) – Present cost

The Choice of NPV Techniques


- When complexities such as tax credits, cash costs, in addition to interest and its deductions
(i.e. cost of bankruptcy proceedings), etc. appear relevant to a company’s financial
decisions, the APV approach may be easier to use.
- The reason for this is that APV treats these cash-flow effects separately, by first estimating
the cash flows, then discounting each one at a rate appropriate to their unique risk.
- On the negative side, APV does not have the automatic characteristic of being consistent
with maintaining an intended ratio between the various kinds of financing (debt / equity) a
company uses.

- 15/56 -
Notations

Cash Flows
FCFt Free cash flow: the amount of cash a company can distribute to its capital suppliers at time t
due to an investment.

It Interest cash flow at time t

Tc Corporate income tax rate

ITSt Interest tax shield cash flow ITSt = It x TC

FCF*t Unleveraged (ungeared) free cash flow: amount of free cash flow a company is expected to
generate at t due to a project, not including income tax shields.
FCF*t = FCFt – ITSt. = All equity CFs,
ie. CFs that would exist if the project was only funded by equity (MCQ4B3)

Market Values
Et Market value of the equity of the investment at time t
Dt Market value of the debt of the investment at time t
Vt Market value of the investment at time t .
Vt = Et + D t

Discount Rates
re Required return on the equity of the investment.
rd Required return on the debt of the investment.
rd* Cost of debt as a rate to the investment
rd* = rd x (1-Tc)
rv Overall weighted average return on the capital claims of the investment.

rv = D (rd) + E (re) = Overall capital return/ cost.


V V
rv* Weighted average cost of capital (WACC) of the investment.

rv* = WACC = D (rd*) + E (re)


V V
ru All-equity unleveraged (ungeared) required return on the investment.

Investment Evaluation Techniques

WACC-NPV
t FCF*t
NPV0 = Σ ---------
t=0
(1+rv*)t

APV
t FCF*t ITSt
APV0 = Σ [ -------- + -------- ]
t=0
(1+ru)t (1+rd)t

- WACC reflects the


i. Financial and operational risk of the project
ii. The interest deductibility involved in the project
The project WACC would reflect the project’s proportional debt and equity rather than the actual amount.
(MCQ4A6)
- 16/56 -
- 17/56 -
Module 5 – Estimating Cash Flows for Investment Projects
- Estimating investment cash flows means that financial managers must keep the following in
mind:

1. Inclusion of all corporate cash flows affected by the investment sometimes means
that financial analysts must invoke the idea of economic opportunity costs.
2. Inclusion of all relevant cash flows means that analysts must include cash flows
from interactions of the investment with other activities of the corporation.
3. Inclusion of all relevant cash flows also means that analysts must know what things
should be omitted from the investments cash flows. I.e. sink costs are to be
ignored. An investment should be discontinued if its future cash flows PV is less
than the company would obtain by selling or abandoning the project now or later.
4. Inclusion of all relevant cash flows means that analysts must be very careful that
the accounting numbers provided for a project are interpreted correctly. I.e.
Overhead costs are typically not indicative of the incremental cash flows that a
project will require. Accounting numbers can include non-cash expenses
(depreciation), and arbitrary activity measures such as floor-space devoted to the
manufacture of a product. It is however correct to include as cash outflows the
changes or increments to overall corporate expenses caused by the acceptance of
the project.

- There are many corporate cash flows that should not be included because they are not
incremental (i.e. managers salaries); they would not be affected by the acceptance or
rejection of a project.

Summary
- All changes that would be caused in the cash flows of a corporation by its accepting a
project must be included in the analysis of the project.
- ONLY cash flows are to be included.
- Estimate CFs as if project were purely equity financed, no debt, so no interest payments.

Operating CF = Operating profit – Change in net Working Capital + Change in Overheads

NWC figure comes from changes in receivables, payables, inventory and cash year on year.

For project budgeting CF analysis, interest payments are not included as these are payments to capital
suppliers. This effect is caught in the discount rate (WACC). [RevEx2(ii)8]

Total CA = Account receivable = Cash outflow in terms of NWC


Total CL = accounts payable = Cash inflow “ “ “ “ [MCQ5A7]

If the cny does not sell the FA at the end of the project, we must include a sum to reflect the benefit the cny gets
from their use. [MCQ5A8]

- 18/56 -
Module 6 – Applications of a Company’s Investment Analysis
- Real asset investment decisions = capital budgeting

The Payback Period


- The number of periods until a project’s cash flows accumulate positively to equal its initial
outlay.
- Companies use this method by picking a maximum period of time beyond which an
investment’s payback will be unacceptable, and rejecting all investment proposals that do
not promise to recoup their initial outlays in that time or less.
- Payback periods and NPV can yield different answers.
- Problems with the payback period include:
a. It ignores all cash flows beyond the maximum acceptable payback period.
b. It does not discount the cash flows within the maximum acceptable period, thereby
giving equal weight to all of them. This is inconsistent with shareholder opportunity
costs.
c. It has too much bias to short term CFs (MCQ6B1)
d. It is not concerned with the profitability of the project (MCQ6B1)
- Some companies have altered their payback period techniques to be discounted payback
periods. Concern ‘a’ above is still valid.
- If a company feels it must use the payback period, a rudimentary estimate of the maximum
allowable period should be set:
1 1
Payback = ------- - ----------------
rv* rv*(1+rv*)n

where n is the number of periods in the projects total lifetime.

- This payback is generally only accurate for projects with fairly consistent cash flows each
period.
- If these restrictions are met, the above will show the number of periods across which, if the
original outlay is not returned (in FCF*), the investment will have a negative NPV.

The Average (Accounting) Return on Investment (AROI)


- Calculates a rate of return on the investment in each period by dividing expected
accounting profits by the net book value of the investments assets.
- These numbers are then added and the sum is divided by the number of periods for which
rates of return have been calculated. The result is then compared to a minimal acceptable
return (often an industry or company average).
- AROI does not discount cash flows and the numbers used are the wrong ones.
- However AROI does have some value as an evaluation of control device to check the
progress of an ongoing project on a period-by-period basis.

Average profit (over the period in question)


AROI = ---------------------------------------------------------------------------------------------- (FAQ +/- AROI)
Average investment (if depreciated to zero it’ll be the initial investment divided by 2)

Average investment = (Initial value + disposable value) ÷ 2

IRR vs. NPV


- When there is an outlay, an inflow, and another outlay (in the form of an opportunity cost),
multiple IRRs can exist – Sign changes across time can yield multiple IRRs.
- A pattern of sign changes can also produce a project that has a totally upward sloping
relationship between NPV and IRR. To correctly accept a project in this case the IRR would
have to be less than the hurdle rate.

- 19/56 -
- There are various means that can be used to make the IRR come up with a correct answer
in a particular situation. The difficulty being that you must know beforehand that you are
going to have a problem with the IRR, and what the solution is.
- Another situation in which the IRR can cause problems is in multiple-period cash flow
investments which require a different discount rate for each cash flow. The cash flows IRR
can still be found, but at which hurdle rate is it compared? The YTM of a security with the
same risk and cash flow patterns as the investment would have to be found.

- IRR assumes that interim CFs are reinvested at the IRR rate.
- NPV “ “ “ “ “ “ “ “ discount “ (MCQ6A8 and Dec 2005Q1.d)

IRR vs. NPV in Mutually Exclusive Investment Decisions


- In situations where multiple investment options must be compared and subsequently
ranked NPV is best.
- When IRR must be used the incremental cash flow analysis technique should be used.
The algorithm steps are:
1. Take any two projects out of the group.
2. Find the one that has the highest net positive cash flow total (sum of all FCF*). The
investment with the highest net cash flow is the defender, the other the challenger.
3. At each time point, subtract the cash flows of the challenger from those of the
defender, the resulting stream are the incremental cash flows.
4. Find the IRR of the incremental cash flows.
5. If the IRR is greater than the appropriate hurdle rate, keep the defender and throw
out the challenger and vice-versa.
6. Pick the next project out of the group and repeat the process using the winner of
step 5 until only one investment remains.
7. Calculate the IRR of the winner. If it is greater than the hurdle rate, accept it;
if not then reject all the projects.
- This algorithm works because it looks at the IRR of choosing one project over another,
instead of each cash projects IRR.
- There are also situations in which the incremental cash flow method of choosing among
investments should never be used:
1. When the incremental cash flows have more than one change of sign across time.
2. When the projects differ in risk or financing, so that they require different hurdle rates.

- IRR is more likely to give incorrect indications of investment ranking among a group of
mutually exclusive investments when the investments:
i. are of significantly different magnitude
ii. “ “ “ “ duration

t0 t1 t2 t3
Project CFs, discount rate 10%: No.1: -$2500 +$1200 +$1300 +$1450
No.2: -$2500 +$1300 +$1300 +$1300

1. Take No.1 and No2.


2. No.1 ΣCF= 1450 > No.2 ΣCF= 1400
No.1 defender No.2 challenger
3. Challenger CFs – Defender CFs
t0 t1 t2 t3
-$2500 +$1200 +$1300 +$1450 Defender
+$2500 -$1300 -$1300 - $1300 Challenger
Incremental CF = 0 -$ 100 0 +$ 150

4. Through trial and error IRR = 22.5% of the incremental CFs


5. 22.5% > hurdle rate of 10% => keep Defender No.1, throw out Challenger No.2
6. No next project
7. IRR of the winner (No.1) = 25.9% > hurdle rate 10% => accept winner
No1 is the better of the 2 projects

The cross-over rate  is where the NPV of the 2 projects is equal  is also the point where the NPV of the incremental CFs
between the projects is equal to zero.

- 20/56 -
The Cost-Benefit Ratio and the Profitability Index

The C-B Ratio


- The ratio between the present value of the cash inflows and the cash outflows of an
investment:

- Where Inflowst + Outflowst = FCF*t


- An investment is accepted if CBR >1, and rejected is CBR <1.
- A CBR >1 would have a positive NPV, and a CBR <1 would have a negative NPV.
- When faced with the question of an investments desirability both CBR and NPV will
produce the same recommendation.
- For mutually exclusive events however, CBR is attracted to those investments that have
the greatest ratio differences between inflow and outflow present values instead of actual
cash or value differences.

The Profitability Index


- The ratio of the accumulated present values of future cash flows to the present cash flow of
an investment.

- The PI is a ratio measure and therefore suffers from the same problems of CBR.
- PI is unsuitable for ranking investments because it displays another relative measure, the
wealth increase per dollar of initial outlay instead of the wealth increase itself.

The CBR and PI techniques of investment analysis give the same indication of investment acceptability which
indication agrees with NPV, if investments are dependent.

Capital Rationing
- The set of methods used to choose a group of projects that will maximise shareholder
wealth while having limited funds available is called capital rationing techniques.
- When having to choose between a few projects, one simply looks at all the possible
combinations of investments that lie within the budget and choose the package with the
greatest NPV. This is called exhaustive enumeration.
- When confronted with many projects to choose from, one common method is to calculate the
profitability indices for the investments, and to list them in declining order of PI. Investments are then accepted in
order of PI, until the budget has been exhausted.
- A project may be skipped because its outlay is too large, and the next one having a small
enough outlay taken as you work down the list.
- The PI technique must be used with some caution in ranking investments when the highest
PI projects do not use up the entire budget.
- Being under capital rationing is an undesirable situation. It can mean that you have not
been able to solve internal organisation/communication problems, or that the capital market
is unconvinced of your prospects. This implies that you will be forced to forego investments
that would have increased the wealth of shareholders.
- The existence of high market required rates should not be interpreted as a capital-rationing
situation. This is simply a signal that your capital costs are also high.
- The capital rationing situation implies that financing beyond the budget constraint carries
not a high but an indefinite cost.

Soft capital rationing  within the organisation


Hard capital rationing  outside the organisation  capital markets
Small high risk firms. Markets not sure about the information out of the cny, discount rate offered too
high. Better info and good track record lead to easing of this problem. (June 2006 Q1.c)
- 21/56 -
Capital rationing tends to occur when cnies have complicated management structures which cause problems in information
flows.

Investment Inter-relatedness
- This is when the acceptance or rejection of one investment affects the expected cash flows
on another.
- Mutual exclusivity is a form of economic inter-relatedness.
- Combinations due to capital-rationing are also inter-related.

Positive Neither Negative

Mutually
Purely Somewhat Somewhat
Independent Exclusive
Contingent Positive Negative

Economic Inter-relatedness of Investment Cash-Flows


- When dealing with economic relatedness among investment proposals companies must
specify all possible combinations of inter-related investments along with their unique cash
flows and NPV. The combination with the highest NPV is chosen.

Renewable Investments
- When companies must choose among investments in real assets where the life span and
cash costs are different for each option the equivalent annual cost technique is used.
- The technique is as follows:
1. The NPV of a single cycle for each asset is found.
2. Divide each NPV by the annuity present value factor for the number of years in
each assets replacement cycle at the appropriate discount rate.
3. The result is the constant annuity outlay per period that has the same NPV as the
asset.
4. Compare the per-period equivalent annuity outlay for each asset and choose the
one with the lowest cost per period.

NPV Annuity PV Factor Cst annual equivalent CFs


Largemouth -$21,461 2.4868* = $8,630 accept
Lunker -$15,678 1.7355* = $9,034 reject
*appendice 2 (Table A2.2) 10% discount rate, 3 periods for Largemouth and 2 for Lunker

2 machines with the same EAC  chose the shorter life cycle ie. life as it would give the
option of updating the technology more quickly.

- 22/56 -
Inflation and Company Investment Decisions
- The real rate of return is the difference between the nominal rate and the expected
influence of inflation on required rates for some time in the future. Because there is no way
to measure such expectations effects, the real rate is not measurable.
- Nominal cash flows and nominal discount rates should be used when dealing with inflation
on corporate investment decisions. If the analysis is performed carefully, the impact of
inflation on the investment, and on shareholder wealth, will appear in the NPV.
- A common error is for cash flows to be stated in real terms, those observable today.
Analysts should always be explicit in requesting inflated future cash flow estimates.
- Another trait is that because many governments require production assets be depreciated
across time, when inflation occurs, the costs of assets will increase across time faster than
the rate of inflation. This results in FCF* increasing at a slower rate than inflation.
- Accelerating depreciation schedules have been put in place to help offset this effect. For
example double-declining balance, or sum-of-the-years digits methods. (June2009Q1d)
- Debt suppliers can be particularly concerned about inflation and required rates. The reason
for this is that debt contracts promise specific amounts of nominal cash at particular times
in the future. If the nominal interest rate that suppliers get at the inception of their
investment turns out to be a poor estimate of actual inflation, debt suppliers will achieve a
real return different from their initial expectations.

NOMINAL means the actual number of $$ that would change hands at the time the purchase is made
REAL price is the number of $$ that would have been exchanged to purchase something before the
inf. took place
Inflation-free return = REAL rate

(1+N) = (1+R) (1+I)

In an inflationary environment, the net effect of inflation on depreciation expenses for CF purpose is
taxes will increase faster than the inflation rate and FCF will increase at a rate less than inflation.

If inflation is going to fall steeply and stay at a low level and market prices do not reflect this
expectation, a cny would replace long term debt with short term debt. As IR will tend to fall with
inflation. [Practice final examination MCQ10]

Leasing
- A contractual agreement between an asset owner (lessor) and a company that will actually
operate the asset without owning it (lessee)
- The most common type of lease is a financial or capital lease – where the lessor is
usually in the business of leasing assets.

The Economics of Leasing


Advantages
- Leasing allows for higher tax benefits than the alternative of borrowing and purchasing an
asset.
- Information asymmetries exist on certain types of assets, and leasing can serve to lower
the costs of such information problems.
- There are economies of scale in the management of specialised asset leasing.
Misconceptions
- Leasing saves money because the lessee does not have to make a large capital outlay to
purchase an asset.
- Lessee debt capacity is higher since they do not need to borrow money to buy the asset.

Evaluating Leases
- Cash flows used would include: cost of purchasing, lost depreciation tax shields, lease
payments, and lease payment tax shields.
- The correct discount rate for performing an NPV is the after tax interest rate (rd*).
- It is important to know what lease rate would allow for a positive NPV when negotiating
lease agreements with a lessor.

- 23/56 -
December 2006 Q1e:
The lease or buy decision is another capital budgeting problem. Workout the PV of the lease CFs and set
these against the cost of actually buying the asset.
Operating CFs irrelevant as they’d be the same purchased or leased.
The lease effectively displaces debt, as debt would be used to finance the purchase, so the disc. rate is
the after-tax cost of debt [rd*= rd (1-Tc)].
Salvage value = complicating factor = outflow for the lease PV, discount rate = cny WACC because it is a
risky CF.

Managing the Investment Process / Biases


- All depends on the quality of the CF forecast. To avoid too rosy a picture hence poor
investment decisions, directly relate management compensation to shareholder wealth
outcomes.

Biases In capital Budgeting (June 2009 Q1c)


- Overconfidence: over-optimistics CFS
- Projects that particular managers want done  overstating everything
- Overly cautions and avoid risky projects
- Too low a discount rate
- Treatment of inflation. Nominal CF only or Real CF only, no mix up
 Reward managers for the amount of wealth created by their projects
EVA analysis.

Using Economic Income Performance Measurement


(EVA = Economic Value Added, EP Economic Profit, etc.)
See June 2007 Q1.d
- Charging a capital cost against the net CFs of a company division in a given period, and
seeing if there’s anything left over. Typically calculated by WACC x invested amount = $
capital cost. If positive left over, revenue covered costs and opportunity costs of its capital
suppliers, leaving and ‘economic profit’ for its shareholders.
- Nothing more than ‘period-by-period’ applications of WACC-NPV.
- EP’s strength is to uncover company operations that are profitable in an accounting sense,
but not in an economic sense.
- ++ EP => positive effect on share value; no ++ EP => declining share value
=> EP tied to shareholder wealth, hence an excellent management performance measure.

- 24/56 -
Module 7 – Risk and Company Investment Decisions
Return SML

Risk

- The security market line or SML describes the relationship between risk and return as
being positive: the higher the risk, the higher the required return. (MCQ7A1)

Risk and Individuals


- To an individual capital supplier, risk is best measured by the standard deviation of rates
on return on the entire portfolio of assets – or by the extent to which possible outcomes are
likely to differ from the mean expected outcome.
- In order to figure out the riskiness of a set of securities one must quote the probabilities of
various rates of return or the probability distributions of returns, for example:
Portfolio probability distribution
Rate of Return Probability
8.5% 35%
11.0% 10%
13.5% 30%
16.0% 25%
- The next step is to calculate the mean return of these probabilities, or:
0.085 x 0.35 = 0.02975 Mean return
+ 0.11 x 0.10 = 0.01100 “
+ 0.135 x 0.30 = 0.04050 “
+ 0.16 x 0.25 = 0.04000 “
Sum 0.12125
Mean = 12.125% = Expected return
- The standard deviation is calculated:
(0.085 - 0.12125)2 x 0.35 = 0.00045992
(0.110 - 0.12125)2 x 0.10 = 0.00001266
(0.135 - 0.12125)2 x 0.30 = 0.00005672
(0.160 - 0.12125)2 x 0.25 = 0.00037539
Sum = 0.00090469 = VARIANCE
√0.00090469 = 0.03008 = Standard deviation
= 3.008% = Standard deviation
- The result is a reflection of the risk inherent in a portfolio.
- Unfortunately, studies to date show that the empirical relationship between risk (measured
as standard deviation of return) and the actual level of return earned is not good.
- In the 1950’s Harry Morowitz was the first to show that company security holders are
indeed risk-averse, and require higher returns when the risk is higher.
- He also showed that the resulting positive relationship between return and standard
deviation of return would only be true for the entire portfolio and not for the individual assets
within.
- This is because part of the standard deviation of return for individual assets is diversified
away when included in a portfolio with others.

A risk averse investor is one who prefers higher returns and lower risk. (MCQ7B4)

- 25/56 -
Risk, Return, and Diversification
- Continuing with the above example:
Return outcome Probability
Asset A 10% 45%
20% 55%
Asset B 7% 65%
12% 35%
- Expected returns per asset are:
A: (0.10 x 0.45) + (0.20 x 0.55) = 0.155 or 15.5%
B: (0.07 x 0.65) + (0.12 x 0.35) = 0.0875 or 8.75%
- Standard deviations are:
A: (0.10 - 0.155)2 x 0.45 = 0.00136 B: (0.07 - 0.0875)2 x 0.65 = 0.00020
(0.20 - 0.155)2 x 0.55 = 0.00111 (0.12 - 0.0875)2 x 0.35 = 0.00037
= 0.00247 = 0.00057
√0.00247 = 0.0497 or 4.97% √0.00057 = 0.0239 or 2.39%

- The logical way to find the risks and returns of the portfolio formed therefore seems to be to
take the average of the returns and standard deviations for the two individual assets:
Avg Return: (0.5 x 0.1550) + (0.5 x 0.0875) = 0.12125 or 12.125%
(same as the portfolios expected rate of return above!)
Avg Std Deviation (0.5 x 0.0497) + (0.5 x 0.0239) = 0.0368 or 3.68%
(differs from the std deviation for the portfolio of 3.008%)
- Obviously the weighted average standard deviation of return of individual assets in the
portfolio is not a correct way to calculate the std deviation of return of the portfolio.
- In order to derive the portfolios return probability distribution, we must know how individual
asset returns interact. This information comes in the form of a joint probability distribution:

- Each cell inside a box describes the probability of a particular set of returns being
simultaneously earned by both assets A and B.
- The joint (interior) probabilities must sum in rows and columns to equal the original
probabilities of the individual security returns while the sum of all cells must equal 100%
(1.0).
Portfolio Events and Probabilities

- The whole portfolio has less risk than the average risk of the securities within it due to
diversification. [see December 2006 Q2.1]
- An easier method for figuring out risk of a portfolio is using the correlation coefficient.

- 26/56 -
The Market Model and Individual Asset Risk
- William Sharpe and John Lentner ascertained the only relevant risk in a market where
everyone understands the benefits of diversification is the undiversifiable or systematic
risk of an asset.

- The reason for a minimum level of risk even in a well-diversified portfolio is that there is a
common correlation present in all securities, and this limits the amount of diversification
possible. This common factor is called the market factor (Riskm).
- The systematic risk of securities is thus based upon the extent to which their returns are
influenced by the market. The actual measure of the undiversifiable risk of a security (j) is:
Systematic Riskj = Std. Deviation of returnj x Correlation of j with the market.
- A security with a correlation close to +1 will have a systematic risk close to its standard
deviation – not much of its risk will be diversifiable.
- A security with low correlation to the market will have much of its risk diversified away when
held in a portfolio with other securities, and thus has a low systematic risk.
- A simple manipulation gives a more commonly used formula:
Betaj(βj) = Standard deviation of returnj x Correlation of j with market
Standard deviation of market return

Or βj = σj ρjm  βj = σj σm ρjm  βj = σjm = Covariance jm


σm σ2m σ2m Variance m

where βj is the beta coefficient for j; σjm is the covariance of j and the market; and σ2m is the
variance of the market.

- Also known as the regression coefficient. Provides the same information as the previous
systematic risk measure, but scaled to the risk of the market as a whole.
- For example, a β of 1.0 indicates that an x percent increase or decrease in the return on
the market is associated with an x percent increase or decrease in the return on that
security, while a β of 1.5 indicates an x percent increase or decrease in the market will
result in a 1.5x return on the security. The β coefficient:

- The steeper the slope (the higher the β coefficient), the greater will the returns on the
security j amplify or gear upward (or downward) the returns on the market portfolio.

- 27/56 -
The more positively assets are related in a portfolio, the less there will be to gain from diversification (MCQ7A5).

As correlation declines, the benefits of diversification increased (MCQ7A5).

A positive value for covariance means that the assets move together, range +1 to -1.

Portfolio risk (s = Standard deviation) is equal to the weighted average (linear combination) of the risks of
individual assets when the individual asset risks are perfectly correlated, ie. corr = +1.0

β coefficient is a measure of the undiversifiable or systematic risk.

The REGRESSION coefficient is produced when a regression analysis is done on two variables. The reg. Coef.
will indicate the steepness of the slope of the line of best fit through the different observation (Figure 7.5) that is
the regression line that runs through the different points, and the β is the reg. coef indicating the steepness of
the relationship between the share and the market.

The Market Model or Security Market Line


- If the financial market sets securities returns based on their risks when held in well
diversified portfolios, systematic risk will be an appropriate measure of risk for individual
assets and securities, and the SML as depicted below will dictate the set of risk-adjusted
returns available in the market:

E(rj)

m SML
E(rm) 

rf

1.0 βj

- Above relates the amount of systematic risk inherent in the returns of a security (its β) to
the returns required on that security by the market. The relationship is positive in that the
higher the systematic risk of j, the higher its required return.
- The SML is located with repect to two important points, the risk-free rate (rf) and the market
portfolios risk-return location (m). m has a return of E(rm) and (by definition) a β of 1.0
- The quantitative relationship between risk and return is:
E(rj) = rf + [ E(rm) – rf ] βj  CAPM formula = rf + β (rm – rf) (7.1)
- The SML based returns are the opportunity costs of capital suppliers of companies, and
thus can form the basis for evaluating company investments.
- These investments must offer returns in excess of the capital supplier’s opportunity costs in
order to be acceptable.

The central idea behind the CAPM is that you get higher return for taking on greater risk. (Dec2006Q2.2)

If investors hold the market portfolio, they will have diversified away all their specific risk and be left
with just the market risk. The measure of a share’s market risk in the portfolio will be its beta.

The reason why a portfolio’s risk cannot be reduced to zero by diversification is that there is a
common factor/ influence (the market) among most investments which causes their returns to be
positively correlated.

- 28/56 -
Using the Capital Asset Pricing Model in Evaluation Company Investment Decisions
- The CAPM or SML is a system that generates required rates of return based upon the
riskinesses of assets.
E(rj)
SML
ReturnA A

WACC  WACC

ReturnB B

βj
βB RISKWACC βA

- The WACC of any company is in fact an average of the risk-adjusted rates of return of the
company’s various endeavours, including its asset types and associated future cash flow
expectations.
- In order to be acceptable, an investment must offer an expected return in excess of the
return depicted on the SML for the investment’s systematic risk level. This means that good
investments would plot above the SML, perpendicularly above their systematic risk.
- In the above example, investment A is above the WACC, thereby implying it is acceptable.
However it is below the SML which indicates that it does not offer a return high enough to
compensate for its risk. Investment B has the opposite problem.
- A company should not generally apply its WACC as an investment criterion. It will only give
a correct answer when an investment’s risk is the same as the average risk of the entire
company. (MCQ7B7)
- Most companies are aware that projects can differ in risk, and that some adjustment of
criterion is advisable. Usually this takes on the form of fixed increments or decrements to
the company’s average criterion.

Estimating Systematic Risk of Company Investments


- To use the SML for estimating required returns, the amount of systematic risk (size of the
coefficient) of a project must be specified. There are several ways to do this:
a. If project is of the same risk as the existing company, and its shares are traded on
the stock market, one can merely look up the β coefficient of the company’s shares
in one of the financial reporting services that supply such data.
b. If risk differs (+ or -) from the company average, the investment may be similar to
another company’s. In such situations, the β coefficient of the other company can
be used. This is also valuable when the shares of the investing company are not
traded, but those of a similar company are, and the investment is simply a scale
change.
- When market generated β coefficients are unavailable, the systematic risk measure must
be constructed artificially. The best approaches to such estimates begin with a β coefficient
for the company / division thinking of undertaking the project, and adjusting that coefficient
for the differences between the project and the company or division.
- In constructing β coefficients from the characteristics of the investment itself, it is necessary
to concentrate upon the underlying factors affecting the returns on the project.

Some Considerations
- If the projects revenues are expected to be quite volatile in reaction to overall market
activity, relative to the divisional / company average, an adjustment to the β coefficient must
be made.
- Similarly, on the cost side, if fixed costs of a project comprise a relatively high proportion of
its total cost, the β coefficient of the project must be adjusted upwards – this is described as
operational gearing. = CF ÷ Total cost (MCQ7B8)

- 29/56 -
Retail operations have high FC investments in their stores, once covered, extra sales
generates high level of profit.
- One preliminary adjustment that must be performed when constructing β. If the beginning
value is from a company that has borrowed money, the value must be purified before the
other adjustments are made. This is done by:

βu = βe E + βd D = weighted average of the 2 β (D+E)


V V

Where βe and βd are observed equity and debt β coefficients, E and D are their
observed market values, and V= E + D.
- Once βu is solved it must be adjusted for revenue risk differentials .
To adjust for revenue risk differential, βu is multiplied by the ratio of the investments
revenue volatility to that of the company:
Revenue-adjusted β = βu Project revenue volatility o
Company revenue volatility

- Next β is adjusted for operational gearing: Fixed Cost Adjustment 


Project βu = Revenue-adjusted β x2 (1 + Project fixed cost %) 2
(1 + Company fixed cost %)

- The final step that remains is to re-adjust the reconstructed and ungeared β coefficient for
any financial gearing planned for the project . In order to do so, we must know the β
coefficient for the debt that will be issued for the project as well as the gearing ratio. With
these two items as well as the ungeared β coefficient of the project, the equity coefficient
can be calculated using:

βu = βe E + βd D
V V

Estimating the WACC of an Investment


- Invoking the SML relationship will allow us to find the return required on the equity of the
project.

E(rj) = rf + [ E(rm) – rf ] βj  CAPM formula = rf + β (rm – rf) (7.1)

β is determined as above
rf is given by government bond interest rates (YTMs) for comparable maturity
investments in such bonds.
E(rm), the expected return on the market portfolio, is a function of the risk-free rate and
therefore it makes more sense to estimate the difference between E(rm) and rf. This
figure is the historical average market (i.e. LSE, NYSE) return above the risk-free rate.

Equity risk premium = Market risk premium = Stock market risk premium = (rm – rf)

- Once the equity required return is determined one can find the after tax cost of capital using
the same equation (if applicable).
- The next step is to find the WACC of the project:
rv* = WACC = D (rd*) + E (re)
V V

The correct time to use a cny β to estimate the discount rate for a project is when the project is small in size
compared to the existing cny. MCQ7A10

- 30/56 -
Other Considerations on Risk and Company Investments

Certainty Equivalents
- It is possible to adjust downward the expected future cash flow itself for its risk
characteristics, creating a certainty-equivalent cash flow, and to discount that cash flow
at the risk-free rate.
- The SML equation must be changed to state cash-flows:
CFce = CF – o E(rm) – rf ox Covariance (CF,rm)
Variance (rm)
- The certainty equivalent cash flow (CFce) is found by subtracting from the expected risky
cash flow (CF) an adjustment for its systematic risk.
- That adjustment uses a variant of the market price of risk, [E(rm) – rf]/ Variance (rm),
multiplied by a measure of the systematic risk of the cash flow, Covariance (CF, rm).
Variance (rm) is the variance (standard deviation2) of the market return, and the covariance
(CF, rm) is the covariance (β coefficient of the CF x variance of the market return) of the
cash flow with the overall market.

Risk Resolution across Time


- A commonly encountered complexity in investment analysis is that the risk of an investment
can be foreseen to change as time passes, yet a decision as to whether to undertake the
investment must be made now.
- A common error in this situation would be to treat the investments entire cash flow set as
having the same risk.
- The basic question is whether to undertake an initial outlay, where the desirability of that
outlay depends upon the outcome of the test. For example in giving up $500 000 now there
is a 50% chance of receiving 0 and a 50% chance of receiving $2 500 000 one period
hence. Expected payoff is thus:
0(0.5) + 2 500 000(0.5) = 1 250 000
- In order to be undesirable, the discounted certainty equivalent of the $1 250 000 would
have to be less than $500 000.

Conclusion
- If shareholders are already well diversified, diversification at the company level is irrelevant
(and probably costly) to them.

- 31/56 -
Module 8 – Company Dividend Policy
- Dividends are the amounts of cash that a company distributes to its shareholders as the
servicing of that type of capital.
- Since any residual cash not paid as dividends is still ‘owned’ by shareholders, this retained
cash is reinvested in the company on behalf of shareholders. The dividend decision is thus
also a cash-retention or reinvestment decision.

Dividend Irrelevancy I
- The net result of changing a company’s dividend is the substitutability of capital gains (i.e.
share value increases) as the dividend is reduced for cash when it is paid.
- Increased dividends = decreased market value, and vice versa.
- The investment / dividend decision:

- The connection is a truly organic one for the company: if dividends are changed, and no
other action undertaken, the company’s investments will also change. Using the above
figure, an increase in dividends would be shown as a widening of the dividend pipe and a
narrowing of the retention pipe resulting in a smaller investment amount.
- To isolate the effect of dividend choices, the company’s investment plans must be kept
intact as dividends change.
- Increase in dividends = increase in new equity (more shares issued)
- Decrease in dividends = decrease in new equity (less shares issued)
- The company share value in total is unchanged, therefore existing shareholder wealth is
the same.
- When the effect of company financial decisions upon shareholders’ portfolios can be
undone by the offsetting actions of shareholders, the company financial decision is
irrelevant.

Dividends and Market Frictions

Taxation of Dividends
- From the shareholders perspective, it is after-tax dividends that are of interest. The
dividend substitute - capital gains, are also potentially liable for taxation.
- Of the two it is usually dividends that are taxed more heavily.
- In countries where dividends are net of company taxes, and the dividends paid are taxed at
the shareholder level, dividend payment is expensive.
- In such a tax system where ‘double taxation’ of dividends is unavoidable, there is a strong
tax incentive against the payment of dividends for companies seeking to please their
shareholders.
- Some countries have imputation systems which impute an amount of company taxes to
shareholders based upon the dividends that companies pay, and then give shareholders a
credit on their taxes for that amount.
- This only balances the double payment effect if personal income tax liabilities of
shareholders = the tax credit.

Dividend taxes are a market friction but not a transaction cost of dividends. MCQ8B4

- 32/56 -
Transactions Costs of Dividend Payments / Preference of shareholders
- Brokerage fees to be paid for transactions.
- Shareholders may prefer one dividend policy to another depending on their preferences for
consuming wealth across time and the costs they would pay to achieve the desired
consumption pattern.

Flotation Costs
- Companies themselves incur costs in raising money from capital markets when they pay
dividends so high as to require new shares to be issued.
These costs are called Flotation Costs.
- Depending on the mechanism of sale, these flotation costs can be significant (5-25% of
total value of issued shares).

Combined Friction’s
- Taxes, transaction costs and flotation costs and information costs net bias in most cases is
against the payment of dividends.
- The resulting optimal dividend policy would be to find all the investments with positive
NPVs and retain as mush cash as is necessary to undertake them; if there is cash left over,
a dividend could be paid; only raise new equity capital when funds insufficient for all
investment – sometimes called a ‘passive residual dividend policy’ (not optimal).

Dividend Clienteles: Irrelevancy II


- Different groups of shareholders have come to be called clienteles in finance. The
interpretation is that they comprise groups that would be willing to pay extra to get the type
of dividend policy that is best suited to their own tax and consumption proclivity.
- It is unlikely however that a company choosing one policy over another will be of benefit to
shareholders because there are likely to be no relatively under-serviced clienteles willing to
pay a premium for the change.
- A better term than irrelevancy is probably ‘inertia’.
- A company switching policies can actually be costly to its existing clientele, so whatever a
company’s current policy is, it is likely to be optimal.

Other Considerations in Dividend Policy

Dividends and Signalling


- Gives indication about future performance of the cny.
- It is in the interests of both managers and shareholders to have share prices reflect new
information (good or bad) as quickly as possible.
- Alterations in dividend policy are a subtle way to communicate this information.
- In order to be truly effective though, dividend payout must be relatively smooth over time.
- Share dividend: instead of dividend, additional partial share of the company is paid.
Preserve cash for investment purpose, can reduce taxes, can increase borrowing capacity,
increasing risk for shareholders.
- Share splitting: a two for one split is the same as a 100% share dividend.
Might be a sign of financial difficulty and this is one way of freeing up cash to repay
bondholders.

A one for two share split (reverse stock split -> raises the share price) means you have half
the number of shares, but they are worth twice as much as before.
Cnies undertake share splits because
1. Their shares have become too expensive
2. It improves their liquidity
3. It signals growth prospects

Liquidity of share = ease and efficiency of trading in the market (financial market liquidity).

Signalling: (i) reverse stock split, (ii) share repurchase, (iii) increase in payout ratio

- 33/56 -
Dividends and Share Repurchase
- Share repurchases are nothing more than a cash dividend to shareholders.
- Company claims of investing in itself are bogus as long as there is one share
outstanding.
- In some countries money received in share repurchase transactions is taxed more lightly or
not at all. Share repurchases on the open market also show signs of being signalling
attempts that receive a positive response from holders.
- One type of share repurchase that is not so positive for shareholders is a targeted share
repurchase. This is a transaction wherein a company offers to repurchase only particular
shares (usually held by potential buyers). The repurchase price is at a significant premium
over the market price.

The traditional view of dividends says that shareholders prefer dividends now with high payout ratios rather than
capital gains (the bird in the hand argument) MCQ81A

The bird in the hand argument is wrong because the riskiness of distant dividends is already accounted for in
the discount rate. MCQ8A3

The bird in the hand argument regards dividends paid now as being more valuable than uncertain capital gains.
FAQ M8

In perfect market environment, if dividends are reduced, market value will be higher because fewer new share
are issured. You release more cash for investment => thus reduce the need for issuing more shares, so any
increase in NPV will be captured by the old shareholders. MCQ8B2

The risky position in the perfect market is reducing dividends, because it means that the shareholders is
actually receiving less cash from the cny and more of his wealth is tied up in share value. MCQ8B3

Important factors to consider when deciding the size of dividends


• Profitability of cny (and prospects for profits over business cycle)
• Cny’s need for additional finance (liquidity position)
• The ability of the cny to raise additional finance on the markets
• Its shareholders dividend preferences
• Gearing level which might limit the cny’s ability to pay dividend

Raising money from the markets is expensive, so the passive policy saves on flotation coasts. The
unpredictability of dividend payments is the strongest argument against passive policy. Shareholders like
predictable dividends. MCQ8A4

Information costs cnies are constrained on what they can tell to investors, so there’s no free flow of information
between the cny and the investor. The cny will favor a dividend policy that is stable and predictable which they
can use the dividend information to the investors. (profiler final exam 1: dividends)

In the real world with market frictions, optimal dividend policy = passive residual dividend policy.

- 34/56 -
Module 9 – Company Capital Structure
- A company’s capital structure is the extent to which it is financed with each of its capital
sources (debt and equity).
- Straight debenture: the company’s assets in general are to be pledged as collateral for the loan. (MCQ 9.5)
It has a floating charge on the cny’s asset, whereas the Secure Debenture has a fixed charge on a specific asset.

Capital Structure, Risk and Capital Costs


- Debt is not more expensive than equity supposedly due to debt carrying required (higher-
priority of senior) interest payments whereas equity does not. It ignores the fact that
dividend payments are legitimate capital costs of the company.
- Debt is not cheaper than equity since the issuance of debt increases the attendant returns
required on equity (often called the implicit cost of borrowing).
- Performing a comparison of EBIT-EPS for two companies enforces this rule.
- Results can be charted on an EBIT-EPS chart. (Figure 9.2 p9/8)
- The differences in the level of steepness in EPS ranges are verbally described as gearing
or leverage (the variability of operating income is amplified). Financial risk.

Capital Structure Irrelevancy I: M & M (Merton Miller and Franco Modigliani)


- The M&M economics of company capital structure predicted that it would make no
difference to shareholder wealth whether the company borrows money or not. Financial
markets will ensure this.
- Because shareholders can borrow and lend on the same basis as companies, any benefit
(or detriment) residing in company borrowing can be duplicated (or canceled) by
shareholders borrowing or lending transactions in their own personal portfolios.
So company capital structure does not matter.
- A wealth increase is impossible since identical future cash flow expectations can be
achieved in a different manner, and less expensively. Shares of any company must sell for
just what it would otherwise cost to acquire the same future cash flow expectations.

Arbitrage and Prices – A Digression


- Arbitrage is a transaction wherein an instantaneous risk-free profit is realized.
- Efficient financial markets abhor arbitrage, and arbitrage opportunities cannot be expected
to exist for any significant time in a market with well-informed investors.
- Market prices must adjust to cause all equivalent future cash flows to sell for the same
price.
- This adjustment occurs naturally with the forces of demand and supply.

Summation of Capital Structure Irrelevancy I


- The M & M ideas make clear that the total value of the company must be unaffected by a
change in its capital structure. E.g.: Capital structure & company values (w/o taxes):

- 35/56 -
Capital Structure and capital cost without taxes
- The below illustrates the behavior of the required rates of return and overall capital cost of
the company: it alters the company’s capital structure.

- M&M argument over capital structure stated that cny’s average cost of capital is not
affected by the level of debt or equity. MCQ9A4

- With respect to specific weighted average relationships determining rv (overall capital


return/ cost), we can imply:

rv = D (rd) + E (re)  rv = D (rd) + (1-D) (re)


V V V V

- The higher proportion of lower-cost debt exactly offsets the lower proportion of higher cost-
equity, such that their weighted average is unchanged (rv=cst).

Capital Structure Decisions and Taxes


- Companies are taxed by the government on the amount of income or profit that they
make.
- Recall interest tax shields: ITS = It x Tc
- When taxes exist, and when interest is deductible by companies, companies will tend to
use debt as their primary source of capital.
- Debt is therefore cheaper in the sense that the total of taxes paid by companies and their
shareholders will be lower than if the companies were to issue equity.

Summation of Capital Structure relevance with Taxes


- Operating cash flows that a company produces are transformed by the taxation system
before they can be claimed by capital suppliers. This transformation is different depending
on the capital structure of the company.
- Because of the tax advantage in company borrowing, a company with debt in its structure
will be more valuable than an otherwise identical company that does not borrow.
- To find the value of this tax benefit (ITS is a stream of cash flows):
VITS = ITS
Rd

- The entire firm could be valued by either debt plus equity or the following APV type
situation (Value unleveraged + leveraging-based tax benefits):

V = VU + VITS

- 36/56 -
- The relationship between changes in capital structure and changes in capital claim and
company values are as follows:

Capital Structure and Company values with taxes

Capital Structure and Capital cost with taxes

- Effects on required returns and capital costs:

The WACC (rv*) is the discount rate that gives the correct valuation when applied to a company’s
ungeared free cash flow (FCF*), not including ITS.

- Notice that the company’s cost of capital (WACC) steadily declines as the company
substitutes debt for equity in its capital structure. This is because even though a company’s
value is increasing, it’s ungeared FCF* must (by definition) be unchanged as D/V
increases. So by the relationship
V = FCF* rv* must be declining as D/V increases.
rv*
- A company’s cost of capital (WACC ie. rv*) is therefore lower the higher its proportion of
debt.

Capital Structure Irrelevancy II: Taxes (June 2007 Q2.3)


- Merton Miller has argued that as more and more borrowing is undertaken by companies in
economies with progressive personal taxes, the interest rates necessary to sell bonds to
high personal-tax investors will increase causing the benefits of company borrowing to
disappear.
- The tax benefits of company borrowing compete with other mechanisms used to reduce
taxes (depreciation, credits) which tends to reduce debts advantages, particularly when the
amounts of income that require shelter from taxes is uncertain.
- Borrowing to obtain tax benefits is less desirable in an uncertain world than when future
‘income’ streams are known with certainty.

Capital Structure and Agency Problems (see module 12 as well)


- Agency deals with situations where the decision-making authority of a principal
(shareholder or bondholder) is delegated to an agent (managers of a company).
- Agency considerations concern themselves with the instances where
conflicts of interest may arise among principals and agents, and how they are resolved.
- One important mechanism used to resolve conflicts of interest is by the issuance of
complex debt contracts. For example, some debt claims carry a convertibility provision.
This means that under certain conditions, at the option of the lender, a bond can be
exchanged for common shares. Therefore avoiding certain agency costs.

- 37/56 -
- Another instance of agency conflict occurs when a company in financial distress is unwilling
to undertake a profitable investment because the resulting effect would be to help
bondholders, not shareholders.

Least helpful in resolving an agency conflict would be secured debentures. It does not stop managers
embarking on risky projects. MCQ9Q6
Vs. restrictive covenants/ convertible bonds/ maintenance of minimal financial ratios.

Convertible bonds, equity warrants, executive share options are issued by the cny so they form part of the
cny capital structure. Equity traded options are an exchange created investment not issued by the cny so
do not form part of the capital structure. MCQ9A1

Equity traded options are the most common type of equity for derivatives. They provide the right, but not
the obligation, to buy (call) or sell (put) a quantity of stock, at a set price (the strike price), within a certain
period of time (prior to the expiration date).

Dividend payments represent CF leaving the cny and going to shareholders. In certain situations
bondholders may be worried that the cash leaving the cny might impair the cny’s ability to service its debt
obligations. As a result clauses in debt contracts limiting dividend payout.

Default and Agency Costs


- The true costs of bankruptcy or financial distress are:
a. Legal costs and administrative costs
The costs involved in pursuing the legal process of realigning the claims on the assets of the company
from those specified in the original borrowing contract.
b. Loss of profits as a result
The implicit and opportunity costs incurred in this effort relative to what would have happened had the
company financed instead by equity capital.
c. Loss of customers
d. Loss of credit
- Unless there is some unique benefit to the issuance of a particular type of claim (such as
the interest deductibility of debt), there is no reason to think that one type of claim will be
better than another.

Magnitude of bankruptcy costs, most to the least:


(a) aeroplane manufacturer, (b) photocopier cny, (c) construction cny, (d) supermarket cny.

Other Agency Considerations


- Perk consumption beyond the point where management productivity is efficiently
enhanced.
- Conglomeration to increase the size and reduce the CF risk of the company, thus
stabilising management remuneration, with no benefit to shareholders holding well
diversified portfolios.

Making the Company Borrowing Decision


- No quantitative method.
- One point that stands out as likely to be of importance to the optimal amount of borrowing
is tax considerations. But the practitioner must be very careful to judge the net tax benefits
of borrowing.
- There is a common notion in practitioner finance that risky business should borrow less (or
be lent less) than companies that are not so risky. This is more of a Darwinian rule of
thumb than a thought out, validated notion. This rule probably works due to agency costs –
risk is likely to make agency costs higher.

The borrowing decision contains signalling power to investors.


By increasing borrowing management is signalling that they have confidence in the strength of
cash flows in the future. MCQ9A9

- 38/56 -
Book Values and Borrowing
- Practitioners argue book values should be used for measuring the extent of company
borrowing while academic types argue that market values are the correct measure.
- The use of book values in the real world makes sense. They are a good measure of the
extent to which values will not be upset by financial distress when the company is engaged
in borrowing.

When using debt and equity weights to calculate the WACC, we use market values – these will
reflect the current required rate of return on the assets, the market’s perception of the risk of
the company. It reflects the value of these assets today. Book Value is likely to be out of date,
may reflect decisions taken years ago and not to be relevant today.

Techniques of Deciding upon Company Capital Structure

1. Examine what companies in similar lines of business have decided about the amounts
they will borrow.
- The best way to do this is to look at company averages for borrowing ratios in the industry
of interest.
- The distance of a company’s debt ratio from the industry average determines the borrowing
decision.
2. Financial planning – a detailed examination of the company’s future cash-flow
expectations (including those associated with the borrowing alternatives under consideration) so as
to decide upon the best choice of financing method.
- The company financial planner, in possession of a capacity to simulate the cash flow and
financial statements of the company across the future, asks a series of what-ifs of the
planning model.
- The result is a set of possible future outcomes for the company under the sets of conditions
and financing alternatives that the planner examines.

Suggestions for Deciding about Capital Structure

1. The company should use simulation to attempt to forecast its cash flows and financial statements
across the foreseeable future under the various alternative proposals for financing.
2. If simulations indicate a significant chance for coming into conflict with covenants of the borrowing
contracts in ways that would damage the operational aspects of the firm, borrowing should be
avoided.
3. If simulations show that tax benefits of borrowing simply replace other tax benefits there is little
reason to borrow.
4. If a company’s value is largely in tangible assets, more borrowing is sustainable.
Industry gearing ratios are useful to see what other companies have been able to sustain.
5. If potential lenders fear company action to the detriment of bondholders, the company should
attach covenants to the bonds to alleviate some of that concern (e.g. convertibility/call provisions).
6. There are also reasons why companies would choose to avoid new equity issuances (e.g. loss of
ownership control) and such considerations may outweigh the negative aspects of borrowing.
7. When a tentative conclusion has been made, see if the result would be inconsistent with the capital
structures of other companies in the same line of business. If so, it should be determined whether
this is an improvement over the usual practice or a signal that something has been left out of the
analysis.

- 39/56 -
Order of preference for financing a cny: 1. retain earning, 2. debt finance, 3. equity finance
1 is the top one because finance is available for use without the scrutiny of financial markets. It is ‘internal’ finance. Debt and
equity = ‘external’ finance as it has to be raised on the capital markets and the cny will face scrutiny because they have to
publish a prospectus detailing the use of the cash. Debt is preferred to equity because there is a tax break on the interest
payments, debt is cheaper. The markets will usually look at a cny that is selling equity as having problems with the debt
markets. This is something of a last resort (the equity market). RevE1Q35.

Because of non-cash items like depreciation it is possible for the cny to have negative income but positive CF. MCQ9B

Which type of high growth cny would be more likely to use debt than equity for finance expansion?
Cnies that may need heavy capital expenditures, eg telecom cnies – they use the debt markets to finance the investment in
new infrastructure – they are profitable and can take advantage of the tax shields available.

High growth cny using debt (are seldom) because they are profitable, generate a lot of CF and can take advantage of the
TSs available (Telecom cnies). RE3Q7.

As one of the tools available to resolve agency problems, the call provision allows shareholders to buy back the bond at a
fixed price. MCQ12A11.

Key factors in the cny borrowing decision (Dec 2008 Q2.1 and Profiler Final Exam CS1)
1. Cny ability to generate CF to service the interest and principal requirements at the appropriate times
2. Ability to fully utilise the tax shields that comes with borrowing
3. The asset backing the cny has that can be offered as collateral in case of default
4. The ability to access the financial markets
5. Likely costs of financial distress
6. Level of borrowing by similar cnies in the same line of business

Reduce high gearing: retire of repay debt. Dividend sacrified to free up cash to pay off debt. Equity issue, Division,
subsidiaries, assets could be sold for cash. Capital expenditure could be cut to release cash for debt repayment.

EBIT = Operating income

Debt requires lower returns than equity because (i) of the contractual nature of interest payments, (ii) of security over assets
when borrowing, (iii) debtholders have a prior claim on operating cash flow. MCQ9A2

Only cny with gearing have financial risk (in real danger, the bondholders could take control of the cny), so all equity cny
have no financial risk, only business or operational risk. Equity is foregiving.

Financial distress will only occur in cnies with borrowing. MCQ9B


When taxes are introduced to the MM argument on CS it gives a greater advantage to debt finance. Cnies do not have
100% levels of debt to equity because:
• They may not have enough income for 100% borrowing to be worthwhile
• Cnies may not be certain about the sustainability of their income. MCQ9B6

A cny is looking to choose between 2 projects, X and Y. The projects will require $20K of debt funding.
Project X payoff probability end of the project: NPV 20K 25%probability NPV 22K 50%prob. NPV 24K 25%prob
Project Y payoff probability end of the project: NPV 16K 25%probability NPV 26K 50%prob. NPV 32K 25%prob
Bond holders would favour X because it will guarantee payments to the bondholders in any outcome. Y has the probability
of greater overall returns. There’s a 20% chance that the bondholders would not get back all their money (16-20= 4 short).
MCQ9B7

- 40/56 -
Module 10 – Working Capital Management Company Capital Structure
- Working capital is the set of balance sheet items that would be included under
Current assets and Current liabilities (WC=CA-CL),
- WC= the net of Cash, Receivables, Inventories and Payables.

- CA = cash, marketable securities, accounts receivable (debtors), and inventories;


CL = accounts payable (creditors), short-term borrowings, and other liabilities coming
due within one year.

Risk, Return, and Term on Investments (Dec. 2007 Q2.1)


- The nature of short-term finance is that it tends to be risky in the sense of requiring the firm
to frequently renew the principal amounts of financing outstanding; this could become a
problem during ‘hard times’.
- The rates of return on financing either short or long term activities are best understood by
considering the costs of the financing type.
- Interest rates are not the reason for return or cost differences between short or long term
finance.
- The costs depend on reversibility differences between the types of finance. In situations
where companies find themselves with unforeseen reductions in the need for financing,
short-term finance is dispensed with (‘reversed’) quickly at the end of its term.
- Therefore short-term finance is less costly than long-term finance and because lower costs
mean higher return, it also exhibits a higher return.
- This is exactly opposite of the risk return characteristic of its assets.

Maturity Matching

Cost Low cost High cost

Combining Risk and Rates of Return on Assets and Financing


- Companies currently face the decision as to the best term structure of assets and financing.
An old rule of thumb is to finance short-term assets with short-term liabilities and long-term
with long-term. This is called maturity matching.
- The result of this is a mixture of risks and returns that is both potentially profitable and
survivable.

Management of Short-term Assets and Financing


- Rather than considering the desirability of each specific short-term asset, managers adopt
policies governing the firm’s investment within each type.

Optimisation and Short-term Investment


- Management techniques attempt to balance costs and benefits in such a way as to produce
the highest net benefit or (equivalently) the lowest net cost of investing in short-term
assets.

- 41/56 -
- Efficient management of asset investments

ST finance is more easy to reverse than LT finance. If you have a mortgage and it is a 10 year
fix and you want to get out of it, it is very expensive. Whereas if you have a two year discount
deal, it’s much cheaper to exit that deal. Once you of a cny has committed to a LT borrowing
arrangement, the lender is expecting that stream of CFs for the 10 years. They will be
disappointed if you try to break that arrangement, so they impose steep penalty fees if you do.
This makes LT finance more costly.

If you have lower costs, as in making any product, you will be able to make higher returns.

ST assets (stock, marketable securities) are less risky than LTA (factories, offices) because
STA will realise cash quickly without much loss in value. MCQ10A2

The costs of ST financing are lower than LTF because STF is more easily cancelled. MCQ10A3

STF is more risky than LTF because it needs to be renewed at least once a year. and there’s a
risk that it may not be renewed. With LTF the cny just needs to make the interest payments.
MCQ10A10

Management of Cash Balances


- As indicated above cash and near cash assets (interest earning bank deposits and short-
term marketable securities) confer the liquidity benefit to companies investing in them.
- In company operational language, liquidity means such assets are used for:
a. Transaction uses of cash - reality that debts must eventually be paid in cash.
b. Precautionary/anticipatory reserves – recognise that there may be events that
cannot be anticipated which require cash, as well as anticipated future cash needs
of major dimensions.
c. Compensating balances – cash amounts contractually left on deposit with banks.
- The costs of cash balances are the transactions costs of switching between higher and
lower interest-bearing securities and accounts, and the differential interest rate earned.
- Management of the process requires that there is enough cash on hand to meet the
transaction, precautionary, anticipatory, and compensating requirements of the company,
while minimising the transaction costs and foregone interest.
- A simple model of cash usage:
Cash Usage
Maximum

Cash Replenishment
Cash
Balances

Minimum

Time

- Assume an interest penalty of i percent (interest foregone) in holding cash balances, and
that each time cash is replenished there is a (fixed) transaction cost of $T. The optimising
of cash replenishment amounts in this situation is solved by:

- 42/56 -
$r = [(2 x $D x $T)/ i ]1/2 (10.1)

where $r is the optimal amount of cash replenishment,


$D is the total annual amount of cash spent by the firm
AKA the economic order quantity.
- A more realistic picture is one where a company’s cash expenditure is lumpy and cash
receipts are more seasonal / cyclical.
- In such a situation the best a manager can do is specify a probability distribution of
potential cash balance changes. The solution is as follows:

upper bound

return point

lower point

- $M is the lower bound, the minimum amount of cash below which the balance is not
allowed to fall, $U is the upper bound; and $R is a return point.
- When cash falls to $M enough interest bearing securities are cashed to return the balance
to $R. When cash balances increase to $U, securities are bought with excess cash to bring
the balance to again $R.
- If $U and $R are well chosen, the costs of maintaining the cash balance are minimised.
The formula below chooses $R:

$R = [(3 x $T x s2) / 4i ]1/3 + $M (10.2)

where $T and i are as above, and s2 is the variance of the changes in cash balances – if the
amount of increase / decrease in cash balances is expected, by the probability distribution, to
be $c for each of the number of times (t) cash balances can change per day,
s2 = $c2 x t.
- $U is part of this solution:
$U = $M +3 ($R - $M) (10.3)

and purchase 2 ($R - $M) of interest-bearing securities


- One note of caution is that technological change has and will continue to relegate much of
the above analysis to little practical significance due to electronic funds transfers.

Management of Receivables
- Companies usually find it necessary to hold accounts receivable and inventory stocks.
- Inventories are handled similarly to cash (but with the cost of shortages considered).
- A higher level of receivables promises more credit sales and thus more customers willing to
purchase, but also portends a longer waits until the actual receipt of cash from a sale and a
higher likelihood of never being paid (bad debts).
- One issue in the investment in receivables is the deterioration in the quality of customer
credit accompanying an increase in the amounts owed to the company. Ways of discerning
who should receive credit:
a. Credit-reporting agencies supply information to a company at a cost.
b. A company’s own records of customer payment histories can yield useful
information about the likelihood of a customer paying.
c. Sophisticated statistical analysis (i.e. discriminant analysis).

- 43/56 -
- At some point rejecting the marginal customer ceases to be worthwhile. This is the point
where incremental expenditure for search and evaluation exceeds the expected gain from
discriminating.
a. If company accepts everybody:
Expected Profit = (# of good customers x Profit per customer)
+ (# of bad customers x Loss per customer)
b. If company performs a credit analysis:
Expected Profit = (# of good customers x profit per customer)
+ (#of bad customers x loss per customer)
– (Cost of credit analyses)
- Caution: a company who accepts everybody must keep it private or else the ratio of bad to
good customers will rise.
- One other approach to the management of receivables is to calculate the NPV associated
with a proposed change in credit terms for a company:
NPV = Change in PV of sales receipts
– Change in variable costs
– Change in working capital investment

Management of Short-term Financing


- Short-term financing is best considered a function of the company’s line of business and
maturity matching. The firm should plan to use short-term financings as required by the
business (lines) it pursues, with the condition that such financings are best done in
association with short-term investments, for balancing of risk and return.
- There are policies that must be set for such a system to be run with optimality.
- One consideration is the extent to which payables (creditors) are managed efficiently as a
separate unit.
- When a firm takes advantage of credit extended by a vendor (known as ‘trade credit’), there
is usually a set of payment conditions associated. Almost always these conditions have a
time when final payment is due, but also a (shorter) time during which payment would
produce a discount from the market price of what has been bought.
- Usually these payment terms are described by a phrase such as 2/10 net 30 which signifies
that there is a 2% discount for payment within 10 days of invoicing and that payment
beyond that is at full market price and is due within 30 days.
- The proper standard to judge when to pay is the cost of financing the money that would be
used to pay early, or the interest rate on such short term borrowing.
- This interest cost is composed of an annualised discount percentage given for early
payment. The formula is:

(10.4)

Cash Budgeting and Short-term Financial Management


- Short-term financial management is best pursued within the context of a company’s cash
budgeting.
- Cash budgeting is the setting-forth of the company’s cash inflows and outflows of cash over
some future time period, usually near-term.
- Without a projection of a company’s cash position, there is no raw data upon which to base
management decisions.
- Without some detailed knowledge of when and how much cash a company needs or has in
excess, there could be embarrassing last-minute plea to the bank for loans, or (worse)
detrimental effects to the business. Excess cash is, on the other hand, destined to be a
low-earning asset.

- 44/56 -
Appendix – Financial and Ratio Analysis
Liquidity Ratios (short term survival) are designed to measure a company’s ability to meet maturing its short
term obligations.

Current Ratio = Current Assets/Current Liabilities >2 o


Where current means turned into or paid out of cash within one year.

Quick Ratio = (Current Assets – Inventory) / Current Liabilities >1 o

Profitability Ratios (long-term survival) measure management’s overall effectiveness.

Profit Margin = Net Profit after Taxes / Sales o

Return on Total Assets = Net Profit after Taxes / Total Assets o

Return on Inventory = Net Profits after Taxes / Inventory o

Return on Owners’ Equity = Net Profit after Taxes / Owners’ Equity o

A favourite technique is the 100% statement where sales are set at 100% and each item is
calculated as a percentage of sales.

Capital Structure Ratios – divided into 2 groups:


a) Examination of the asset structure of the company

Fixed to Current Asset Ratio = Fixed Assets / Current Assets o

b) Analyse of the financing arrangement of the company’s total assets,


particularly debt. This group of ratios is known as gearing (leverage) ratios.
Gearing ratios measure the contributions of shareholders with the financing provided by the
company’s creditors and other providers of loan capital.
Companies with low gearing ratios have less risk when the economy goes into a recession,
but also have lower returns when the economy recovers. Companies with high gearing
ratios experience the opposite.

Debt Ratio (Gearing) = Total Debt / Total Assets o

Times Interest Earned = (Profit before Taxes + Interest Charges) / Interest Charges o

Efficiency Ratios (activity or turnover ratios) – how effectively a company has been managing its assets.

Inventory Turnover = Sales / Inventory o

Average Collection Period = Debtors / Sales per Day o

Fixed Asset Turnover = Sales / Fixed Assets o

Financial Analysis and Internal Accounting: an Integrated Approach

The chart approach to financial analysis combines the activity and profitability ratios with the
detailed costs and revenues obtainable from internal accounting systems.

- 45/56 -
Module 11 – International Financial Management

The Foreign Exchange Markets

Exchange Rates and the Law of one Price


- The same thing cannot sell for different prices at the same time.
- Exchange rates portray relationships in wealth exchanges across national borders in much
the same manner as interest rates portray wealth exchanges across time (future). They
generate the expectation of purchasing power parity across countries (now).
- Some frictions to purchasing power equilibrium include transaction and information costs,
as well as positive and negative impediments to free trade imposed by governments.

The price of the goods should be the same once you adjust for the currency, this is the
PPP theorem. It does not hold in the short term, it is a longer term predictor of FOREX
rates.

PPPs are not always in line with their indicated value because of (i) transaction costs (ii)
physical limitations (iii) government intervention (iii) information costs. MCQ11A4

Spot and Forward Exchange Rates


- Spot rate is the basis on which a given currency is exchanged into another on that date.
- A forward rate is the going price for exchanging between currencies at some future time.
- By entering into a forward exchange contract, a trader commits to purchase or sell an
amount of currency at a fixed price and time in the future.
- The buyers and sellers of forward exchange contracts are companies seeking to avoid the
risk of exchange rate fluctuations, or to hedge such transactions.
- The forward exchange market can also be used to speculate in exchange rates; this is
when one commits to purchase £ with $ but has no future $ inflow expectations.
- If a given currency is selling at a higher forward rate than spot rate it is said there is a
forward premium on it, if lower it is called a forward discount.

Exchange Rates and Interest Rates


- A company could accomplish the same expectation as hedging through borrowing funds in
the required currency at an existing interest rate, and exchange those funds for the local
currency at the existing exchange rate.
- If carefully calculated, the exact amount of dollars required so as to pay off the loan with the
cash expected from collecting receivables at a future time point:
Amount borrowed = $ receivables / (1 + existing interest rate)
- This loan amount is then switched to local currency at the spot rate. The local currency is
then invested for the duration of the loan at the local interest rate.
- The receivables, when received, are then used to pay off the original loan.
- The cash proceeds to the company would be exactly the same as if the company had
purchased the foreign funds in the forward exchange market.
- This is a necessity of foreign exchange and interest rate markets, due to interest rate parity.
- Interest rate parity ensures that borrowing or lending in one currency at the interest rate
applying to that currency will produce the same final wealth as borrowing or lending in any
other currency at its interest rate.
- Interest rates must adjust to ensure such parity or there will be arbitrage opportunities.
The following relationship must hold:
Relative interest rate = Relative forward exchange discount or Premium

- 46/56 -
FORN (Foreign currency), DOM (Domestic currency), SR (Spot Rate), IR (Interest Rate).
The DOM is always the single unit, and the FORN is so many per one unit of DOM. £0.6266/$1
£ = FORN, $ = DOM
$1.5960/£1 $ = FORN, £ = DOM

FORWARD Rate (FORN/1DOM) = (1+FORN IR)n x SR (FORN/1DOM)


(1+DOM IR)n

eg. UK IR= 4%, US IR= 2%, SR= $1.45/£1, what is the 1 year forward rate?
The $ is the FORN, the US IR is the FORN IR, and the £ is the DOM unit.
FORWARD = (1.02/1.04) x 1.45 = 1.422

It works both for the PPP (purchasing power parity) and the IRP (interest rate parity).

Expected SRn = (1+FORN infl)n x SR0 (FORN/1DOM) [RevE3.19]


(1+DOM infl)n

The current Japanese-US spot exchange rate is 0.00813 $/¥. The one year interest rate in
Japan is 1%, US is 3.5%. What is the 3 month forward exchange rate?

(1+FORN IR)n = FORWARD Rate (FORN/1DOM)  (1.035)0.25 = Forward Rate


(1+DOM IR)n SR (FORN/1DOM) (1.01)0.25 0.0813

3 month forward rate = 0.00818

(1+RFUS FORN) = (1+WACCUS FORN)


(1+RFUK DOM) (1+WACCUK DOM)

Forward Exchange, Interest Rates, and Inflation Expectations


- Inflation (or more precisely differential inflation) influences exchange and interest rate
markets.
- The key to remember is that inflation in a given currency affects the future purchasing
power of that currency.
- An important determinant of the forward exchange rate between any two currencies is the
expectation of differential inflation rates in the two currencies.
- In purely domestic financial markets, when money is lent the return one expects to receive
is usually stated in nominal terms, or the actual figure you expect to receive.
- What is not guaranteed is what the money will buy. If inflation is expected to exist during
the loan period, the real (stated in terms of purchasing power) return is expected to be less
than the nominal return. Calculated by:
Nominal interest rate = Real interest rate + Effect of inflation
n
(1+ nominal rate) = (1+ real rate)n x (1+ inflation rate)n
where n is the number of periods in question

(1+ N)n = (1+ R)n (1+ I)n


- Since money can be kept in any currency it is reasonable to expect that purchasing power
parity across time as well as across currencies will be maintained by the foreign
exchange market.

Relative inflation = Relative forward exchange discount or premium

R£ = N£ = I£ = FRv UK infl rate = Forward Rate £/$  (1.0475)1/2*** =


0.6337
R$ N$ I$ SR£/$ US infl rate Spot Rate £/$ (1.0240)1/2*** 0.6266

*** 6 month FR (1 year period ÷ 2 for 6 months)

- 47/56 -
- It is not surprising then that the ratio of the forward exchange rate to the spot exchange rate
mirrors the ratio of expected inflation rates in the two currencies.
- Interest rate differentials are caused by inflation differentials, which are the root cause of
the observed discount or premium on forward exchange.

International Financial Management

Hedging International Cash Flow


- Management of a company’s foreign exchange exposure must comprehend the
net exposure in each currency based upon a detailed comprehension of all monetary
accounts.
- Some counter-arguments to hedging exchange exposure are:
a. Real assets in other countries will experience nominal increases in value as
inflation increases and exchange rates move down in that currency.
b. There are significant transaction costs to hedging.
- Much of the complexity in deciding on and tracking the results of hedging is amenable to
automation.
- A newer security is the foreign exchange option. Allows the holder to sell (buy) foreign
currency in the future, but also allows the holder to choose not to.
- The cost of options are usually higher than a simple forward contract since the seller has
weighed very carefully the odds of losing money upon the options exercise.
- The purpose of the transaction plays a role: a forward contract to a currency hedges
exchange risk exactly, whereas an option does not hedge, it actually creates a position that
insures against a bad turn in exchange rates but retains the advantage of a beneficial
movement. Option benefits = costs => no reason to use exchange options!

Investing in Foreign Real Assets


- The process of deciding upon the financial viability of a foreign investment is the same as a
domestic one: estimate expected cash flows and discount at the investment’s cost of
capital.
- One important question is when currency translation should be done in the evaluation of an
investment.
- The process is to discount the future foreign currency cash-flow estimates at the foreign
currency discount rate and translate these foreign currency values into domestic currency
values using the spot exchange rate.

Hedging involves using forwards and futures which lock in a price


Insuring involves using options.

You nee to borrow $1m in 3 months, considerable uncertainty over IR. Take out IR option.
It benefits you if IR went down, protecting you if IR went up. Forwards and futures lock you
in.

- Another issue is that of adjusting for the risks entailed in foreign investments per se. The
tenets for financial managers to keep in mind are:
a. Remember the benefits of diversification – if a company’s shareholders are not well
diversified across international borders, a foreign investment may deserve a lower
risk profile than a purely domestic one assuming the foreign investments cash
flows are not well correlated with a comparable domestic one. If they are well
diversified, the consideration is neutral.
b. Remember the relative uncertainties of the investment – alterations in foreign trade
laws, exchange restrictions, asset confiscation, and friction in repatriation can
increase the risk of a foreign investment. A good analysis of these issues would
wish to consider these relative to comparable domestic risks and add a foreign risk
premium only if truly deserved.

Financial Sources for Foreign Investment


- There would be no real reason to choose one financing location over another based upon
interest rate differences (government subsidies are the exception).

- 48/56 -
- If an exchange rate changes due to inflation in the foreign country, the domestic currency
value of a real asset held in that foreign currency will not necessarily change, because the
exchange rate effect upon value will be offset by the inflationary effect upon value.
- Generally, real assets (plant, equipment, real estate…) held in other countries tend to
experience increases in value as inflation increases, whereas monetary assets do not and
therefore they are serious candidates for the hedging of exchange risk.
- Multinational firms need not necessarily finance their real-asset investments with liabilities
in the same currency as the real asset.
- There is also a truly international capital market from which companies can borrow.
In it, long-term funds are usually called Eurobonds, whereas short-term borrowings are
called Eurocurrency.
- Often borrowing rates are slightly lower in Eurobond and Eurocurrency markets than in
comparable domestic loans, due to lower regulatory costs.
- Repatriation of funds is often a problem for companies whose shareholders are foreigners.
Some countries have significant measures in place that seek to keep profits earned by
foreign firms within their borders. Such things as management fees, royalties, loan interest
and principal repayments, and transfer payments are used to repatriate funds to the
domestic parent company.

Financial Solutions to other International Investment Risks


- There are ‘moral hazard’ risks engendered by dealing with foreign authorities.
- Management of this risk is possible through invoking lessons taught in agency theory. The
trick is to anticipate potential situations where the host country would be likely to take
action, and build automatic and irreversible counter-incentives into the original agreement.
- Usually involves a third party, whom the host country must appease (i.e. World Bank, IMF).

Which of the following is not a risk in international operations?


(a) Government regulation, (b) manufacturing locations (c) capital structure decision (d) dividend decision
The dividend is least exposed to risk in international operations. MCQ11A2

- 49/56 -
Module 12 – Advanced Topics: Options, Agency, Derivatives, and Financial Engineering
Options
- Options are 'contingent claims' because the payoffs to an option depend on what
happens to another value or cash flow often of another security.
- A call option allows one to purchase ('call') another security or asset at a fixed price for a
fixed period of time.
- The shares that can be purchased are known as the underlying assets for the option.
- The price the option allows you to purchase the shares is called a striking or exercise price.
The exercise price is the price that you can buy the underlying asset at. MCQ12A1
- The final date you can exercise the option is known as the expiration date.
- With respect to when options can be exercised there are two types:
European, which can be exercised only at expiration;
American, which can be exercised at any time before or at expiration.
- When an option can be exercised profitably it is said to be in the money, when it cannot it
is out of the money.
- On option owner (holder) need not exercise the option if s/he chooses not to do so.
- The issuer of the option is often termed the option writer. Rarely are options ever
exercised in the sense of shares trading hands, usually its just money. This minimizes
transaction costs.
- If the writer actually owns the underlying securities, the option is called a covered option
or a covered call, otherwise it is a naked option/call.
- If the value of the underlying security is less than or equal to the striking price the exercise
value of the option is zero.
- Another type of option is a put. Allows the holder to sell something (shares) at a fixed price
for a fixed period of time.
- Puts have positive exercise value to the holder when the underlying asset’s value is less
than the striking price.
- Other combinations of puts/calls are formed (spreads, strips, straddles, hedges, butterflies)
in complex transactions so as to generate a particular risk-return exposure to the holder or
writer.

If you buy an option for 20p and the exercise price is 100p and the share price is 90p, this
means that you have paid 20p for the option to purchase shares for 100p ea. These options will
become more valuable as the share price rises.

An option can have a positive value and a zero value. MCQ12A2


Put option holder purchases the option for a ‘premium’. MCQ12A3
The covered put holder has a ST pessimistic view. MCQ12A5
The naked put holder has a ST pessimistic view. MCQ12A6

An Introduction to Option Valuation


- As long as there is some chance that an option could have some exercise value prior to
expiration, the at the money option would sell for a price greater than zero.
- The same argument would apply to out of the money options, though the market price is
likely to be lower since the underlying security must increase more in value.
- In the money options also sell for more than their exercise values as the option holder
stands to gain at exercise the benefit of possible interim increases in underlying asset or
security value, but is not at risk for all possible reductions in underlying asset value.
- Therefore option market value is always above exercise value. The amount by which is
called the option’s premium.

Option’s premium = difference between the exercise value of the option and its market price.
= Market price – exercise price MCQ12A8

If you own a call option on a share, the X= 650p, share value is 700p, the correct description of the
option is ‘in-the-money’ and is worth more than 50p. As long as the option has time left to expiry, its
value will be greater than 50p. MCQ12A7

The issuer of a call option has a SHORT position and expects the asset to fall in price. MCQ12B1
Someone who SHORTS an asset wants a price fall and someone who is LONG expects
the price to rise.
- 50/56 -
Exercise value (intrinsic value) S0 – X for calls >0  in the money
X – S0 for puts >0  in the money

E A Exercise price
B Option premium (above lowest possible value)
C Lowest possible value for option (no premium, no time value)
D Share price
E Option value
C
B Case study 12.1
A D

Market and Exercise value


Exercise value of put option

Option market value


Exercise
price

Out of the money

Exercice value

Value of underlying Value of underlying


Stricking asset asset
Exercise
or
price
exercise price

______________________ Exercice Value of a put option

Calculating the Value of a Simple Option


- Assume price changes are binomial, they can take only one of two values.
- For example a stock trading at $1.50 has a 60% chance of increasing to $3.00, and a 40%
chance of decreasing to $0.75. Graphically shown as:

q = 0.6 uSo = 2.0 x $1.50


= $3.00

So = $1.50

dSo = 0.5 x $1.50


1-q = 0.4
0.6 = $0.75

Where So = current price of underlying security


q = likelihood of the underlying security price increase
uSo = underlying security increased price result
dSo = underlying security decreased price result
u = upward multiplier for underlying security price
d = downward multiplier for underlying security price

- Now consider the value of the in the money option. First step is to specify the final payoffs
to the holder contingent upon what happens to the shares.
Co = current market value of the option
Cu = option payoff at expiration if underlying security price is up
Cd = option payoff at expiration if underlying security price is down
X = exercise or striking price of the option
Cu = max (0, uSo–X)
q = 0.6
= max (0, 3.00-1.25)
= 1.75
Co
Cd = max (0, dSo–X)
1-q = 0.4 = max (0, 0.75-1.25)
0.6 =0

- 51/56 -
C
B

A D
- By invoking the law of one price we can discover Co by calculating the value of another
investment that offers the same future cash flow expectations (AKA a call equivalent
portfolio). This can be found by forming a portfolio of the underlying shares, in combination
with borrowing or lending money. Three more pieces of notation:
rf = risk-free interest rate
Y = the number of underlying shares to purchase
Z = the amount lending (if +) or borrowing (if -) at the risk-free rate

- The payoff if underlying shares increase becomes: YuSo + Z(1 + rf) = Cu = Pu


- If shares decrease it becomes: YdSo + Z(1 + rf) = Cd = Pd

Cu – Cd
- Hence: Y = ------------- (12.1) Formula sheet
So (u – d)

uCd – dCu
- Hence: Z = ------------------- (12.2) Formula sheet
(u – d) (1 + rf)n

if period = 6 months  n = 0.5

- Therefore the value of the option is equal to the cost of the call equivalent portfolio, or:
Co = YSo + Z (12.3) Formula sheet

+ YuS0 + Z(1+rf)
q = 0.6
= 0.7778 (2.0) (1.50) + (-0.5303 (1.10))
= 1.75

Portfolio
+ YS0 + Z + YdSo + Z(1+rf)
1-q = 0.4 = 0.7778 (0.5) (1.50) + (-0.5303 (1.10))
0.6 =0

The option market price is made up of 2 elements: its intrinsic value and its time value. The intrinsic value (or
exercise value) of an option is the value you will receive now if you exercise your option. This is (S0 – X). The rest
of the option’s market value is time value.

Example:
S0= 95, X= 100, Option market price = 5, intrinsic value (S0 – X)= 0 (option cannot have a negative value). Time value = 5
If the share price was 105, and option’s market price = 9, then intrinsic value = 5, time value = 4

The time value = the price you are willing to pay in the expectation of something favorable happening to the share price

TIME VALUE = OPTION PRICE – INTRINSIC VALUE


(my cost) (premium) (exercise value)

Time value = zero at expiry. The highest ‘at the money’. MCQ12B3

You buy Tesco June 350p call option for 9.5p today, with the share price at 345p. What will be the yearly profit/
loss at expiry if the shares are close at 355p at the expiry of the option series?
 Option worth (S0 – X)= 355 – 350 = 5p, you paid 9.5p, you loose 4.5p

- 52/56 -
Valuing More Realistic Options
- Any realistic model must allow for multiple prices, given what can happen to the price of an
underlying security.
- The model above can be changed to cover periods rather than prices.
- The Black-Scholes option model is essentially the same as the 'many short period'
binomial model, except that the time-periods are continuous.
Co = SoN(d1) – Xe−rfT N(d2)  Co = YSo + Z (12.7) Formula sheet
Where
d1 = [ln(So/X) + rf T]/σ(T1/2) + 0.5σ(T1/2) (12.8) Formula sheet
And
d2 = d1 – σ(T1/2) (12.9) Formula sheet
- Interpretation as follows:
Co: calculate the value of a ‘call equivalent’ portfolio.
SoN(d1) = number of shares of the underlying security to be purchased.
where N(d1) is the equivalent of Y in the binominal process
= instruction to take the value of the cumulative normal unit distribution at the point d1.
Xe−rfT N(d2) = amount of borrowing, Z in the binominal process
N(d1) and N(d2) = cumulative normal distribution value of variable d1 and d2.
Xe-rfT = exercise price multiplied by e-rfT
e-rfT = continuously compounded interest or discount rate with e being the base of a
natural logarithm (2.718)
rf = risk-free interest rate
T = a new variable representing the time remaining until option expiry.
ln = natural log of the bracketed expression.
σ = standard deviation of underlying security price.
- There are five variables for determining option value; So, X, rf, σ, and T.
- So and X identify the exercise value of the option, and how far in or out of the
money the option is located. Recall that 'at the money' options carry the highest
premiums.
So = Value of the cny assets
X = Level of interest payments and principal = Exercise price X
- rf determines how much money must be set aside so as to exercise the option
at expiration; as interest rates increase, the present value of the future outlay to
exercise declines, and therefore option value increases.
σ is also positively related to option value. Any random movement in the underlying
security price is more likely to help than harm option value.
σ = Variability of earnings
(An option will tend to follow upward movements in the underlying security price more
so than downward movements.)
As the variance of the underlying asset increases, a call option on that asset becomes
more valuable because in a call option higher underlying asset variance means positive
returns are accentuated and negative returns are dampened (MCQ12.1).
- As T increases, option value increases; reflecting the higher odds that the
security will experience some increase in price, the longer the time until maturity.
T = maturity of the Bond

The most price sensitive of the Black-Scholes variables is volatility. Increases in other variables
(Time to expiry) (interest) have limited impact on the option price, but a rise in volatility will drive
the option rapidly higher.

Put-Call Parity Relationship


Put + Underlying Asset = Call + Riskless PV of Exercise Price (12.10)

P + S0 = C0 + X -  P = C0+ X -– S0
(1+rf) (1+rf)

- 53/56 -
The strategy represented is put option purchase
The bar under the horizontal axis represents the option premium
as the price falls, the put option gains in value.

MCQ12B2

Payoff diagram of a call option seller/ writer

Payoff diagram of a call option buyer


Strike
price Area of gain June 2008 Q1C

Area of loss

Applications of Option Valuation


- The equity of a firm with debt in its capital structure is actually a call option if interest and
principal are paid to creditors, shareholders own the underlying assets of the firm; if interest
and principal are defaulted, creditors will end up with the assets. (June 2008 Q1e)
- Adapting the discussion of the variables in the Black-Sholes option model to the equity of
the geared firm:
- So/X ratio where S0 = underlying asset value = total value of the
company’s assets, X = exercise price = interest and principal promises to
creditors.
If the So/X ratio is high, the option is well in the money, which implies a low proportion
of debt in the company’s capital structure. If low the company has lots of debt.
- An increase in σ means that the operating assets of the firm are more risky.
- Corporate equity claims are also actually put options in the sense that the shareholders can
sell the assets of the company to the bondholders for the lesser of the amount owed on the
debt (the interest and principal) or the assets themselves (defaulting on the loan, and
declaring bankruptcy).
- The law of conservation of investment value means that increases in value have to come
from somewhere.
- Specific actions taken by geared companies can shift wealth from debt holders to
shareholders, leading to conflicts of interest and the theory of agency.

Insurance policies are put options


Deposit on new house is like a call option

When considering the equity in a geared cny as an option


- The longer the time to expiry, the more valuable the option is, ie. the longer the maturity of
the debt, the more valuable is the equity
- The more volatile an asset is, the more valuable an option is
the more variable the earnings of a cny, the more valuable the equity will be
- The higher the interest rate, the more valuable the equity will be
the higher the IR, the less value the debt has in PV terms, so the higher the value of the
option

- 54/56 -
Real Options and ‘Strategic Finance’
- When an investment proposal carries with it and option to alter, curtail or extend a project’s
cash flows at some future time, classic NPV is an inadequate evaluation technique.
- Option imbedded in real asset decisions = real options.

Real options are options to alter, abandon, or extend a project’s cash flows at some future
point. Because of the nature of real options, conventional capital budgeting is not appropriate
for their evalutation. (June 2008 Q1d)

The value for the options would then be added to the basic NPV of the project to give a time
indication of the NPV of the project.

Examples of a Real Option Decision


- The company buys a call option on the Cellular Internet Devices project if it undertakes
beforehand WalkPhone, both currently having a negative NPV.
- A project can either be operated or liquidated at various points during its projected lifetime.
At each of the potential liquidation points there is effectively a put option that allows the
company to sell the project instead of operating it for its remaining lifetime. The firm could
choose to liquidate the project if the exercise price (the liquidation value) is greater than the
present value of the remaining project cash flows plus the value of the additional options to
liquidate further in the future. Hence NPV’s associated with operating the project through its
lifetime must be augmented with the put option values inherent in the ability to abandon or
liquidate the project. Ignoring abandonment or liquidation options can produce misleading
investment NPVs.

Agency relationships
- An agent is an individual, group or organisation to whom a principal has designated
decision-making authority. In the context of the shareholder-bondholder conflict,
shareholders (or their proxies, the financial managers of the firm) are the agents, and the
bondholders are the principals. Principals are those who feel the ultimate effects of the
decisions taken by the agents.
- Agency problem between a firm’s managers and its shareholders, where the former’s
personal best interests conflict with those of the latter.
- Many economists think that an efficient market for company takeovers is an important
solution to the agency problem of manager-shareholder conflict.
- The outsiders are telling the shareholders that existing management is inefficient, and thus
the increase in the company value that will appear upon more efficient new managers
taking over can be allocated between old an new shareholders.
- Any solution to an agency problem requires that there be an overall gain in solving the
problem, which is allocated in such a way that the agent has an incentive to participate in
the solution.
- Agency problem can arise when information asymmetries exist. It occurs when a buyer
and seller in a transaction may have different amount or quality of information about the
decision at hand.
- Then the company issues a call provision (= call option of the bond) that allows the firm to
repurchase the bond from the bondholders at a set price for a given period of time.
- The agency problem caused by the information asymmetry is solved by a complex security,
the call-provisioned bond.

Most likely to help resolve shareholder/ manager agency problems is the threat of takeover. MCQ10A11

Covenants in the bond contract


- positive covenants: maintenance of certain financial ratios.
certain level of operating profit ÷ interest charges (fix charge coverage).
Failure  breach covenants  Bondholder action
- negative covenants: restrictions on dividends, repurchase shares, debt, disposal of assets.

Put provision on contract: allow bondholders to demand repayment. RevE2iQ9

- 55/56 -
The form of finance which would exert the strongest control by shareholders over manager is selling more
shares because it exposes the cny to more scrutiny of investment bankers, brokers, credit rating
agencies, investment analysts and shareholders. MCQ12B10.

Derivatives
- A ‘derivative’ is any financial security whose return or outcome set is derived from some
other asset’s value or return outcome.
- Even simple derivatives can be very dangerous when misapplied.

Participation in Derivative Markets


- There should be careful control and oversight of those responsible for committing firms to
positions in derivative markets.
- Someone in the organisation should understand enough about these markets to appreciate
the risks inherent in a proposed commitment.
- The potential benefits of participating in derivative markets are very large for
many organisations.
- The most common type of transaction is derivative markets is the hedging of risks.
- Properly used, derivatives serve to reduce risk.

The Types of Derivatives


- Interest Rate: forward contracts, futures contracts, options, swaps.
- Stock Market: futures contracts on market indexes, options on market indexes, options on individual securities.
- Foreign Exchange: forward contracts, futures contracts, options, swaps.
- Mortgage: complex derivatives.
- Real Asset: forward contracts, futures contracts, options contracts.
- Hybrids and Exotics

Swaps
- Swaps are derivatives designed to allow hedging the risks of interest rate and foreign
exchange-rate movements.
- Where one party exchanges a stream of cash flow with a counter-party, who provides the
other stream of cash flow to be exchanged.

Exotics
- Exotics are true derivatives, but formulated from combinations or mixtures of other types of
derivatives.
- They are tailored to the very specific risk exposures of a single firm, and can be very
complicated.
- A security that paid off on the contingency that some type of interest rate increased above
a particular cap rate, or declined below a particular floor rate (the distance between the two
being termed a wedding band).

Financial Engineering
- Closely intertwined with the ideas and markets of derivative securities and risk hedging.
- It is the nuts and bolts of designing a hybrid or exotic financial securities to fit very specific
risk-shaping intentions of a firm.

The Elements of Financial Engineering


- Embodies the notion that there are a few elemental building blocks or financial contracts
that can be combined in a rigorous fashion to produce an almost unlimited variety of
non-standard cash flow expectations.
- These building blocks can be classified as having essential characteristics of:
a. Credit extension (such as loans, bonds, etc.)
b. Price fixing (such as futures / forward contracts)
c. Price Insurance (such as call / put options)
- The invention of new or ‘hybrid’ financial securities to fit exactly some requirement of an
individual financial market participant is a matter of combining these elements into a
package of claims that will produce a profile of cash flow expectations meeting this specific
need.
- 56/56 -

Das könnte Ihnen auch gefallen